Net Explanations

  • Book Solutions
  • State Boards

Case Study Questions Class 9 Science Work and Energy

Case study questions class 9 science chapter 11 work and energy.

CBSE Class 9 Case Study Questions Science Work and Energy. Important Case Study Questions for Class 9 Exam. Here we have arranged some Important Case Base Questions for students who are searching for Paragraph Based Questions Work and Energy.

At Case Study Questions there will given a Paragraph. In where some Questions will made on that respective Case Based Study. There will various types of marks will given 1 marks, 2 marks, 3 marks or 4 marks.

CBSE Case Study Questions Class 9 Science – Work and Energy

(1 ) Work done by force acting on an object is equal to the magnitude of the force multiplied by the distance moved in the direction of the force. Work has only magnitude and no direction. Work done is negative when the force acts opposite to the direction of displacement. Work done is positive when the force is in the direction of displacement.The unit of work is newton-metre (N m)or joule (J).

(i) Work done is

(a) Scalar quantity

(b) Vector quantity

(c) Tensor quantity

(d) None of these

(ii) When force acts against the direction of displacement then work done will be

(a) positive

(b) negative

(c) both a and b can possible

(iii) SI unit of work is

(a) Joule(J)

(b) Newton meter(N-m)

(c) both a and b

(iv)You are lifting stone from floor. Work is done by theforce exerted by you on the stone. Theobject moves upwards. The force youexerted is in the direction ofdisplacement. However, there is theforce of gravity acting on the object. Which one of these forces is doingpositive work?

 Which one is doing negative work?

(v) Define 1J of work.

Answer key-1

(iv) Here work done by you is positive work as work is being done in the direction of displacement unlike in case of gravitational force which acts in downward direction against the direction of displacement which is in upward direction.

(v) When 1 Newton of force acts on body and body displaces from its position by 1 meter then the work done is said to be 1 joule (J).

(2) A moving object can do work. An object moving faster can do more work than an identical object moving relatively slow. A moving bullet, blowing wind, a rotating wheel, a speeding stone can do work. How does a bullet pierce the target? How does the wind move the blades of a windmill? Objects in motion possess energy. We call this energy kinetic energy.

Thus, the kinetic energy possessed by an object of mass, m and moving with a uniform velocity, v is

KE = ½ *mv 2

The energy possessed by an object is thus measured in terms of its capacity of doing work. The unit of energy is, therefore, the same as that of work, that is, joule (J).

(i) Energy possessed by body which is in motion is called

(a) Potential energy

(b) Kinetic energy

(c) Nuclear energy

(ii) Which of the following has same unit?

(a) Potential energy and Force

(b) Kinetic energy and work

(c) Both a and b

(iii) Kinetic energy depends

(a) Inversely on velocity of body

(b) Directly on square of velocity of body

(c) Directly on  velocity of body

(iv) Define kinetic energy of body. Give its SI unit

(v) Is kinetic energy scalar or vector? Justify your answer

Answer key-2

(iv) Energy possessed by object due to its motion is called as kinetic energy. Its SI unit is N-m or Joule(J).

(v) kinetic energy is scalar quantity as it is a work done and work done is scalar quantity hence kinetic energy is also scalar quantity and doesn’t have any direction.

(3) Lift an object through a certain height. The object can now do work. It begins to fall when released. This implies that it has acquired some energy. If raised to a greater height it can do more work and hence possesses more energy. From where did it get the energy? In the above situations, the energy gets stored due to the work done on the object. The energy transferred to an object is storedas potential energy if it is not used to cause a change in the velocity or speed of the object.An object increases its energy when raisedthrough a height. This is because work isdone on it against gravity while it is being raised. The energy present in such an objectis the gravitational potential energy.The gravitational potential energy of anobject at a point above the ground is definedas the work done in raising it from the ground by height h

to that point against gravity.Let the work done on the object against gravity beW. That is,

work done, W = force × displacement

Therefore potential energy (PE)= mg*h.

(i) Energy possessed by body due to its position is called

(ii) SI unit of potential energy is

(iii)You do work while winding the key of a toy car. The energy transferred to the spring inside is stored as

(iv)Find the energy possessed by an object of mass 5kg when it is at a height of 10 m above the ground. Given, g = 9.8 m/s 2 .

(v)Find the work done by Gravity on an object of mass 5 kg which moves from height 10m to ground when it is released from height of 10 m. Given, g = 9.8 m/s 2 .

Answer key-3

(iv) we have potential energy as

=5 ×9.8 ×10

(v) work done, W = force × displacement

= 5 ×9.8 ×10

(4) The form of energy can be changed from one form to another. What happens to the totalenergy of a system during or after the process?Whenever energy gets transformed, the totalenergy remains unchanged. This is the law ofconservation of energy. According to this law, energy can only be converted from one form to another it can neither be created nor destroyed. The total energy before and after the transformation remains the same.The lawof conservation of energy is valid inall situations and for all kinds of transformations. Thus during motion the sum of the potential energy and kinetic energy of the object would be the same at all points. That is, potential energy + kinetic energy = constant.Andcalled as mechanical energy.

(i) Which of the energy conversion occur in electric iron?

(a) Electric energy converted into heat energy

(b) Electric energy converted into light energy

(c) Heat energy converted into electrical energy

(ii) When ball drops from height which of the energy conversion takes place

(a) Gravitational potential energy converted into kinetic energy

(b) Kinetic energy converted into Gravitational potential  energy

(iii) When ball is thrown vertically upward which of the following quantity remains constant?

(c) Potential energy

(iv) State law of conservation of energy.

(v) In hydroelectric power plant which energy conversion happens?

Answer key-4

(iv) This is the law ofconservation of energy. According to this law,energy can only be converted from one form to another it can neither be created nor destroyed. The total energy before and after the transformation remains the same.

(v)  In hydroelectric power plant potential energy of water reservoir is converted into electric energy.

(5 ) A more powerful vehiclewould complete a journey in a shorter timethan a less powerful one. We talk of the powerof machines like motorbikes and motorcars.The speed with which these vehicles changeenergy or do work is a basis for theirclassification. Power measures the speed ofwork done, that is, how fast or slow work isdone. Power is defined as the rate of doingwork or the rate of transfer of energy. If anagent does a work W in time t, then power isgiven by

P= work/time

P= W/T. The unit of power is watt.

(i) The rate of doing work is defined as

(ii) Total energy consumed divided by total time taken is called as

(a) Average power

(b) Instantaneous power

(iii) Let A and B having same weight start climbing the rope and reach height of 10m. Let A takes 10sec while B takes 12sec then work done

(a) By both will be same

(b) By A is more than work done by B

(c) By B is more than work done by A

(iv) Define 1 Watt of power

(v) An electric bulb of 20W is used for 5h per day. Calculate the ‘units’ of energy consumed in one day by the bulb.

Answer key-5

(iv) A power is said to be 1 watt when 1 joule of work is done within 1 second of time.

(v) Power of electric bulb = 20 W

Time used, t = 5 h

Energy = power × time taken

= 0.02kW × 5 h

= 0.10 kW h

= 0.10‘units’.

The energy consumed by the bulb 0.10 units

Jaru mitaya

Leave a Reply Cancel reply

Your email address will not be published. Required fields are marked *

Save my name, email, and website in this browser for the next time I comment.

We have a strong team of experienced Teachers who are here to solve all your exam preparation doubts

Duff and dutt question bunch class 10 the passing away of bapu page 199 to 205, rs aggarwal class 8 math first chapter rational numbers exercise 1c solution, new learning composite mathematics class 5 s.k. gupta anubhuti gangal volume chapter 18a solution, new learning composite mathematics class 5 s.k. gupta anubhuti gangal volume chapter 18b solution.

Sign in to your account

Username or Email Address

Remember Me

Study Rate

Class 9 Science Case Study Questions Chapter 11 Work and Energy

  • Post author: studyrate
  • Post published:
  • Post category: class 9th
  • Post comments: 1 Comment

Case study Questions in Class 9 Science Chapter 11  are very important to solve for your exam. Class 9 Science Chapter 11 Class 9 Science Case Study Questions have been prepared for the latest exam pattern. You can check your knowledge by solving case study-based questions for Class 9 Science Chapter 11 Work and Energy

Join our Telegram Channel, there you will get various e-books for CBSE 2024 Boards exams for Class 9th, 10th, 11th, and 12th.

Download Books for Boards

In CBSE Class 9 Science Paper, Students will have to answer some questions based on Assertion and Reason. There will be a few questions based on case studies and passage-based as well. In that, a paragraph will be given, and then the MCQ questions based on it will be asked.

Work and Energy Case Study Questions With Answers

Here, we have provided case-based/passage-based questions for Class 9 Science  Chapter 11 Work and Energy

Case Study/Passage-Based Questions

Case Study 1: The figure shows a watch glass embedded in clay. A tiny spherical ball is placed at edge B at a height h above the center A

case study based questions class 9 physics work and energy

The kinetic energy of the ball, when it reaches point A is (a) zero (b) maximum (c) minimum (d) can’t say

Answer: (b) maximum

The ball comes to rest because of (a) frictional force (b) gravitational force (c) both (a) and (b) (d) none of these

Answer: (c) both (a) and (b)

The energy possessed by the ball at point C is (a) potential energy (b) kinetic energy (c) both potential and kinetic energy (d) heat energy.

Answer: (a) potential energy

Case Study 2: The principle of conservation of energy states that the energy in a system can neither be created nor be destroyed. It can only be transformed from one form to another, but the total energy of the system remains constant. Conservation of electrical energy to various forms or vice versa along with devices is illustrated in the figure given below.

case study based questions class 9 physics work and energy

Water stored in a dam possesses (a) no energy (b) electrical energy (c) kinetic energy (d) potential energy.

Answer: (d) potential energy.

A battery lights a bulb. Describe the energy changes involved in the process. (a) Chemical energy →Light energy → Electrical energy (b) Electrical energy → Chemical energy → Electrical energy (c) Chemical energy → Electrical energy → Light energy (d) None of these.

Answer: (c) Chemical energy → Electrical energy → Light energy

Name a machine that transforms muscular energy into useful mechanical work. (a) A microphone (b) Bicycle (c) Electric torch (d) An electric bell

Answer: (b) Bicycle

A body is falling from a height of h. After it has fallen a height h/2 , it will possess (a) only potential energy (b) only kinetic energy (c) half potential and half kinetic energy (d) more kinetic and less potential energy.

Answer: (c) half potential and half kinetic energy

Case Study 3: An elevator weighing 500 kg is to be lifted up at a constant velocity of 0.4 m s –1 . For this purpose, a motor with the required horsepower is used

The power of the motor is (a) 1940 W (b) 1950 W (c) 1960 W (d) 1970 W

Answer: (c) 1960 W

The power of motor in hp is (a) 2.33 (b) 2.43 (c) 2.53 (d) 2.63

Answer: (d) 2.63

Case Study 4: Work and energy are fundamental concepts in physics that help us understand the physical world and the processes happening around us. Work is done when a force is applied to an object, and the object moves in the direction of the applied force. It is calculated as the product of force and displacement. The unit of work is joule (J). Energy, on the other hand, is the ability to do work. It exists in different forms, such as kinetic energy, potential energy, and various other forms like thermal energy, electrical energy, and chemical energy. The law of conservation of energy states that energy cannot be created or destroyed, but it can be transformed from one form to another. Understanding the concepts of work and energy helps us analyze the efficiency of machines, calculate the amount of work done, and comprehend various physical phenomena.

When is work considered to be done on an object? a) When a force is applied to the object b) When the object moves in the direction of the applied force c) When the object remains stationary d) When the object changes its shape Answer: b) When the object moves in the direction of the applied force

How is work calculated? a) Force multiplied by velocity b) Force multiplied by acceleration c) Force multiplied by displacement d) Force divided by time Answer: c) Force multiplied by displacement

What is the unit of work? a) Newton (N) b) Meter (m) c) Joule (J) d) Watt (W) Answer: c) Joule (J)

What is energy? a) The ability to do work b) The force applied to an object c) The distance traveled by an object d) The mass of an object Answer: a) The ability to do work

According to the law of conservation of energy, what happens to energy? a) It can be created b) It can be destroyed c) It can be transformed from one form to another d) It remains constant Answer: c) It can be transformed from one form to another

Hope the information shed above regarding Case Study and Passage Based Questions for Class 9 Science Chapter 11 Work and Energy with Answers Pdf free download has been useful to an extent. If you have any other queries about CBSE Class 9 Science Work and Energy Case Study and Passage Based Questions with Answers, feel free to comment below so that we can revert back to us at the earliest possible By Team Study Rate

You Might Also Like

Cbse class 9 social science(sst) term 1 mcq questions with answers pdf download, class 9 science mcq questions for chapter 15 improvement in food resources with answers, best reference books for class 9th, this post has one comment.

' src=

where did you get this question from pls send the source

Leave a Reply Cancel reply

Save my name, email, and website in this browser for the next time I comment.

Talk to our experts

1800-120-456-456

  • Important Questions for CBSE Class 9 Science Chapter 11 - Work and Energy

ffImage

CBSE Class 9 Science Chapter-11 Important Questions - Free PDF Download

Work and energy comprise one of the most fundamental units of physics. This chapter also sets the tone for the +2 syllabus. So, in this section, we will be dealing with the important questions of chapter Work and Energy Class 9 . We further aim to acknowledge children with all the binding terms, keeping in mind about the previous year questions.

Vedantu can help students to get hands-on the most essential topics from exam perspective. It also offers a widespread platform to know one’s ability through online interrogation from experts. You can also download free PDF for class 9 Science chapter 11 important questions. 

Register Online for Class 9 Science tuition on Vedantu.com to score more marks in your examination. Also download Class 9 Maths NCERT Solutions to help you to revise complete syllabus and score more marks in your examinations.

Download CBSE Class 9 Science Important Questions 2024-25 PDF

Also, check CBSE Class 9 Science Important Questions for other chapters:

centre-image

Study Important Questions for Class 9 Science Chapter 11 – Work and Energy

1 Marks Questions

1. What is the power of a lamp which consumes 1000 J of electrical energy in 10 s?

Ans: Power = Work/Time = 1000Joules/10seconds = 100 Watts

2. Can an object displace without applying force?

3. What is the SI unit of power?

(a) $\mathrm{J} / \mathrm{s}$

(c) $\mathrm{s} / \mathrm{J}$

(d) $\mathrm{J} / \mathrm{s}^{2}$

Ans: (a) $\mathrm{J} / \mathrm{s}$

4. What will be the change in kinetic energy of a body if it starts from the rest?

Positive  

Negative  

It can be positive or negative, depending on the body mass.

Ans: (a) Positive.

5. Which of the following sets of quantities have similar units? 

(a) Power and Energy 

(b) Work and Power

(c) Energy and Work

(d) None of the above 

Ans: (c) Energy and work

6. A body is present at a height ‘h’. Which type of energy will it possess?

(a) Kinetic energy 

(b) Potential energy 

(c) Both kinetic and potential energy

(d) None 

Ans: (b) Potential energy

7. What will be the work done, if a body moves in the opposite direction of the direction in which force is applied?

(a) Positive 

(b) Negative 

(c) Zero 

(d) Cannot predicted

Ans: (b) Negative

8. What will be the work done, if the force is applied at an angle $\theta ?$

(a) $\quad W=F S \operatorname{Cos} \theta$ where $F=$ Force

(b) $\quad W=F S \theta$ where $S=$ Distance

(c) $W=F S \operatorname{Sin} \theta$ where $W=$ work

(d) None of the above

Ans: (a) $W=F S \cos \theta$ where $F=$ Force

9. A body having mass of 5 kg is lifted vertically upto the distance of 9 meters. What will be the work done?

(a) 450J 

(b) -450J 

(c) 45J 

(d) 540J 

Ans: (a) 450J

10. What is the relation between joules (J) and ergs (erg)?

(a) $\quad 1 J=10^{7} \mathrm{erg}$

(b) 1 erg $=10^{7} J$

(c) $1 J=10^{-7} e r g$

Ans: (a) $1 J=10^{7}$ erg

2 Marks Questions

1. When the work is said to be done?

Ans: When a force acts on an object and moves it in the same direction that of force then work is said to be done. 

2. What will be the expression for the work done when a force acts on an object in the direction of its motion.

Ans: Work done = Force × Displacement

If W is the work done, F is the force applied on object and d is the displacement, then the expression of work done will be

3. Explain 1 joule of work done.

Ans: When a force of 1 N (Newton) is applied on an object and that object displaces upto a distance of 1 m (meter) in the same direction of its displacement, then 1 joule (J) of work is done on the object.

4. How much work is done in ploughing a 15 m long field when a pair of bullocks applies a force of 140 N on the plough?

Ans: Since Work done (W) = Force (F) × Displacement (d)

Hence, Work done in ploughing (W) = 140 N × 15 m =  2100 J

5. The force acting on the object is 7 N, and the displacement of the object occurs in the direction of the force is 8 m. Suppose that force acts on the object through displacement, then how much work was done in this case?

Ans:   As we know, Work done (W) = Force (F) × Displacement (d)

Thus, Work done in the given case (W) = 7 N × 8 m =  56 J

6. Define kinetic energy of an object.

Ans: The kinetic energy of an object is a kind of mechanical energy that exists in the object due to its state of motion (movement).

7. Write down the kinetic energy expression of an object.

Ans: If m is the mass of an moving object and v is its velocity, then the expression of its kinetic energy (KE) will be

$K.E=\frac{1}{2}mv^2$

8. Define power.

Ans: The rate by which work is done refers to power. It is expressed by P.

Power = Work done/Time

9. What is 1 watt of power?

Ans: When an object is doing work at the rate of 1 J/s, then the power of that body or object is 1 watt (where watt is the unit of power).

10. An object is thrown at an angle to the ground, moves along a curve and falls back to the ground. The start and end points of the object path are on the same horizontal line. How much work is done by the gravity on that object?  

Ans: There must be a displacement to calculate the work, but since the vertical displacement in this case is zero (because the start and end points are on the same horizontal line), the work done by gravity is zero.

11. How does the state of energy get changed when a battery lights up a bulb?

Ans: The chemical energy of the battery is converted into heat and light energy of the bulb in the given case.

12. Calculate the work done by the force that changes the velocity of a moving body from 5 ms -1 to 2 ms -1 . The body has a mass of 20 kg.

Ans: Since work done by force = Change in the kinetic energy of the moving body

Therefore, Work done by force = $\frac{1}{2}\times m(v_{1}^{2}-v_{2}^{2})$

=$\frac{1}{2}\times 20\times (5^2-2^2)$ =10×(25−4)=10×21 =210 J

13. An object having 10 kg weight is moved from point A to point B on the table. If the distance between A and B is horizontal, what work does gravity do to the object?  Give the reason for the answer. 

Ans: Since the work done by gravity on the object depends on the change in the vertical height of the object, the vertical height of the object will not change. Because the connection level of A and B is at the same height, the work done is zero.

14. The potential energy of an object decreases gradually in a free fall. How does this violate the law of conservation of energy? 

Ans: This does not violate the law of conservation of energy, because the potential energy of an object in free fall gradually decreases with gradual changes until the kinetic energy of the object maintains the state of free fall, that is, the total energy of the object remains conserved.

15. What energy conversion occurs when riding a bicycle?  

Ans: Our muscle energy is converted into mechanical energy while riding a bicycle.

16. Does energy transfer occur when you push a huge rock with all your strength without moving it? Where did the energy you applied go?  

Ans: As long as you push a big rock with all your strength and do not move it, energy transfer will not occur, because cell energy is only used for muscle contraction and relaxation, and also for releasing heat (sweating).

17. A household uses 250 units of energy in a month. How much energy is used  by that house in joules?

Ans: Energy consumption by a house = 250 kWh

Since, 1 kWh = 3.6× 10 6 J hence, 250kWh=250×3.6× 10 6 =9× 10 8 J

18. The output power of the electric heater is 1500 watts. How much energy does it consume in 10 hours?  

Ans: Power of electric heater (p) = 1500W = 1.5kW

Energy = Power × Time = 1.5kW × 10 hours = 15 kWh

19. An object of mass m moves at a constant speed v. How much work does the subject need to do to make it stable?  

Ans: For an object to be stationary, the work done must be equal to the kinetic energy of the moving object.

The kinetic energy of any object is equal to

$K.E=\frac{1}{2}mv^2$ , where m is the mass of the body and v is its velocity.

20. Sony said that even if different forces act on the object, the acceleration of the object can be zero. Do you agree with her, if yes, why?  

Ans: Yes, we agree with Soni, because the displacement of an object becomes zero when many balancing forces act on that object.

21. Calculate the energy (in kilowatt hours) consumed by four 500 W devices in 10 hours.

Ans: Since, Energy = Power × Time 

Hence, Energy consumed by four 500 W devices in 10 hours = 4 × 500 × 10 = 20000 Wh = 20 kWh

22. Free-falling objects will eventually stop when they hit the ground. What will happen to their kinetic energy? 

Ans: The object will eventually stop after it hits the ground in free fall, because its kinetic energy will be transferred to the ground when it hits the ground.

23. A large force acting on an object, and the displacement of that object is zero, what will be the work done?  

Ans: The work done on the body is defined as the force exerted on the body that causes a net displacement of the body.  

Work done = Force x Displacement  

If the force does not cause any displacement, the work done to the object is zero.

24. Write some differences between kinetic and potential energy.

Ans: Differences between kinetic and potential energy:

25. Describe the law of conservation of energy. 

Ans: The law of conservation of energy says that:

Energy cannot be produced or destroyed. It can only be transformed from one form to another.  

The energy of the universe is constant.

26. A person weighing 50 kg climbs the stairs with a height difference of 5 meters, within 4 seconds.  

What kind of work is done by that person?  

What is the average power of that person?

Ans: Mass of the man = 50 Kg 

Distance moved by that man = 5 meter 

Time taken to cover the given distance = 4s

Work Done = Force Acceleration

In this case, the increase in Potential energy = Work done =Mgh =50×10×5 =2500 J

Power =  work Done   Time Taken  = 2500 4 =625 Watts

27. Write differences between power and energy.

Ans: Differences between power and energy are given below:

28. Write down the expressions for

Potential energy of an object

Kinetic energy of an object

Ans: (a) The expression for Potential energy of an object = P.E = mgh 

Where, m = Mass of Body 

g = Acceleration due to gravity 

h = Height 

(b) The expression for Kinetic energy of an object = 1 2 m v 2

Where, m = Mass of body 

v = Velocity of body

29. If a force of 12.5 N is applied to complete a work of 100 J, what is the distance covered by the force?

Ans:   W = Work = 100 J 

F = Force = 12.5 N 

And S is the distance moved or displacement 

Since, Work done = Force Displacement

W=FS 100=12.5× S 100×10 12.5 =S 1000 125 =S 8 m=S (Displacement)

30. A car weighing 1800 kg is moving at a speed of 30 m/s when braking. If the average braking force is 6000 N, it is determined that the vehicle has traveled to a standstill distance. What is the distance at which it becomes stable?

Ans: M = Mass of the car = 1800 Kg 

V = Velocity of the car = 30 m/s 

F = Force applied while braking = 6000 N 

KE= 1 2 m v 2

KE = 1 2 1800×900

KE=810000 J

KE of car = Work done by the car = Force Displacement 

810000=6000× Displacement

810000 6000 = Displacement

135 m= Displacement

3 Marks Questions

1. The kinetic energy of an object with mass m moving at a speed of 5 m per second is 25 J. If its speed doubles, what is its kinetic energy? What is its kinetic energy when its speed triples?

Ans: K.E. of the object = $\frac{1}{2}\times m\times (5)^2$

$25=\frac{1}{2}\times m\times 25$

$m=(25\times y^2)/25=2kg$

If velocity is doubled,

$K.E=\frac{1}{2}\times 2\times 10^2=200/2=100J$ i.e. K.E. will become four times

If velocity is increased three times

$K.E=\frac{1}{2}\times 2\times 15^2=225J$ i.e. K.E. will become nine times.

2. What do you understand about average power?

Ans: The agent may not always be able to complete the same amount of work in a given time period. In other words, the power of this work will change over time. Therefore, in this case, we can take the average power of the work done by the body per unit time (that is, the total energy consumed divided by the total time).

3. Take a look at the steps below. Based on your understanding of the word "work", prove whether the work will proceed.  

Suma swims in the pond.  

The donkey carries a heavy load.  

The windmill draws water from the well.  

Green plants perform photosynthesis.  

The trains are pulled by engines. 

Drying food grains in the sun.  

Sailing boats are powered by wind.

Ans: The work is said to be done when a force acts on an object and moves in the direction of the force. According to this explanation, the following activities were taken in which work will be proceeded:

4. An object weighing 40 kg rises to a height of 5 m above the ground. What is its potential energy? If you let an object fall, find the kinetic energy when it is in the middle.

Ans: Potential energy of the object $=P . E=m g h=40 \times 10 \times 5=2000$ Joules 

Height at which objeot is present when it is in the middle $=2.5 \mathrm{~m}$

As the object is thrown from the rest, hence, its initial velocity $=0$

Since $v^{2}=u^{2}+2 g h$

$ \begin{array}{l} v^{2}=0+2 \times 10 \times 2.5 \\ v^{2}=50 \\ \text { Kinetic energy }=\dfrac{1}{2} \times m \times v^{2} \\ K E=\dfrac{1}{2} \times 40 \times 50=1000 \mathrm{~J} \end{array} $

5. A satellite is moving around the earth. What will be the work done by the force of gravity on that satellite? Give justification.

Ans: The displacement made by the object is perpendicular to the force direction as it is moving on a round path.

$ \begin{array}{l} \theta=90^{\circ} \\ W=F \times s \operatorname{Cos} \theta \\ W=F \times s \operatorname{Cos} 90^{\circ} \end{array} $

$W=F \times 0=0$

Therefore, work done is zero.

6. A person will feel tired if he puts a bundle of hay on his head for 30 minutes. What will be the work done by the person? Prove your answer.

Ans: When a person lifts a bundle of hay above their head for 30 minutes and feels tired, they exert an upward force, and the bundle of hay moves forward perpendicular to the direction of the applied force, so the displacement is zero.

$W=F \times s \operatorname{Cos} \theta$

$ \begin{aligned} W &=F \times s \operatorname{Cos} 90^{\circ} \\ W &=F \times 0=0 \end{aligned} $

Hence, no work done.

7. The law of conservation of energy is explained by discussing the energy changes that occur when we move the pendulum laterally and swing it. Why does the pendulum eventually stop? What happens to the energy and does it violate energy conservation law?

Ans: Bob will eventually stop due to the friction created by the air and the rigid support that holds the thread in place. This does not violate the law of conservation of energy, because mechanical energy can be converted into another unusable form of energy for some useful work. This energy loss is called energy dissipation.

8. How much work is done to stop a car of weight 1500 kg moving with a velocity 60 km/h?

Ans: Given that, initial velocity of a car 

= $60 \mathrm{~km} / \mathrm{h}=(60 \times 1000) / 60 \times 60=50 / 3 \mathrm{~ms}^{-1}$

The object is stopped, thus, its final velocity $=0$

Initial kinetic energy $=\dfrac{1}{2} \times m \times v^{2}$

$K E=\dfrac{1}{2} \times 1500 \times(50 / 3)^{2}=208333.30 J$

Final kinetic energy $=\dfrac{1}{2} \times 1500 \times 0=0$

Therefore, work done $=$ change in kinetic energy $=208333.30-0=208333.30 J$

9. In each of the following cases, the force F acts on an object of mass m. The direction of the object's movement is from west to east and is indicated by the longest arrow. Check the given diagram carefully to see if the work done by the force is negative, positive, or zero.

The force F acts on an object of mass m

Ans: (i) Since the displacement in the first figure is perpendicular to the direction of the force, the work done is zero.  

(ii) Since the displacement in the second figure is in the direction of the force, the work done is positive.  

(iii) Since the displacement in the third diagram is opposite to the applied force, the work done is negative.

10. In the given force-displacement plot, calculate the work done in the time interval of

0 < x < 2 m 

2 < x < 6 m 

0 < x < 9 m 

Force-displacement plot

Ans: The area of force-displacement plot gives the work done, therefore,

a) For time interval of $0<x<2 \mathrm{~m}$

Work Done $=$ Area of triangle OAE

$ W=\dfrac{1}{2} \times \text { Base } \times \text { Height } $

$W=\dfrac{1}{2} \times O E \times A E$

$W=\dfrac{1}{2} \times 2 \times 8$

$W=8 \mathrm{~J}$

b) For time interval of $2<x<6 \mathrm{~m}$

Work Done = Area of rectangle ABED

$W=$ Length $\times$ Breadth

$W=A B \times B D$

$W=4 \times 8$

c) Fortime interval of $0<x<9 \mathrm{~m}$

Work Done $=$ Area of triangle $\mathrm{DBC}$ 

$\mathrm{W}=\dfrac{1}{2} \times \mathrm{B}$ $\times$ Height 

$W=\dfrac{1}{2} \times D C B D$ 

$W=\dfrac{1}{2} \times 3 \times 8$ 

11. Derive the expression of the kinetic energy of an object. Calculate the kinetic energy of a 5 kg object moving at a speed of 2.5 ms -1 .

Ans: The kinetic energy of the body is defined as energy with the dignity of body movement. 

An object with mass m is at rest. The force F N acting on it will cause acceleration ms -2 , assuming the velocity is v ms -1 and covering the distance s m. 

Now from the third equation of motion

$ \begin{array}{l} v^{2}=u^{2}+2 a s \\ v^{2}=0+2 a s \\ 0=\dfrac{v^{2}}{2 s} \\ (u=0 \because \text { body starts from rest }) \end{array} $

From Newton's second law,

$F=\dfrac{m \times v^{2}}{2 s}=\dfrac{m v^{2}}{2 s}$ 

Work Done on the moving Body = Kinetic energy 

$W=$ Force $\times$ Distance 

$W=\dfrac{m v^{2}}{2 s} \times s=\dfrac{1}{2} m v^{2}$ 

Mass of the body $=5 \mathrm{Kg}$ 

Velocity of the body $=2.5 \mathrm{~m} /\mathrm{s}$ 

$K E=\dfrac{1}{2} m v^{2}$ 

$K E=\dfrac{1}{2} \times 5 \times(2.5)^{2}$ 

$K E=\dfrac{3125}{2 \times 100}$ 

$K E=\dfrac{1562.5}{100}$ 

12. A stone is thrown with a velocity of 40 m/s in upward direction.

The potential and kinetic energy of that stone will be equal at what height?

If the stone’s mass is 10 kg, what will be its potential energy?

Ans: Given that, the initial velocity of stone $=u=0$

And its final velocity $=v=40 \mathrm{~m} / \mathrm{s}$

Let the mass of the body $=M$

(a) Kinetic energy of the body $=\dfrac{1}{2} m v^{2}$

And its potentialenergy $=\mathrm{Mgh}$

Now, $K E=P E$

$ \begin{array}{l} \dfrac{1}{2} m v^{2}=m g h \\ \dfrac{1}{2} m \times(40)^{2} m \times g \times h \\ \dfrac{1600}{2}=g h \\ 800=g h \end{array} $ $ \begin{array}{l} \dfrac{800}{10}=h \\ 80 m=h \end{array} $

(b) $\quad P E=m g h$

P E=10 \times 10 \times 80=8000 \mathrm{~J}

13. A body having mass 5 kg and constant velocity 12 m/s is lifted upwards. Calculate:

Force applied in lifting the body

Work done in lifting the body

What will happen to the work done?

Ans: Given that, mass of the body $=m=5 K g$

And height upto which it lifted upwards $=h=12 \mathrm{~m}$.

$\mathrm{g}=$ Acceleration due to gravity $=10 \mathrm{~m} / \mathrm{s}^{2}$

a) $\mathrm{PE}=\mathrm{mgh}$

$P E=5 \times 12 \times 10$ $P E=600 \mathrm{~J}$

b) Force $=$ ?

Work done = Potential energy of the Body

Force $\times$ Distance Moved $=600$

$F \times 12=60$ $F=50 \mathrm{~N}$

c) The work done is stored as the potential energy while lifting the body.

14. Get the expression of the potential energy of an object. Calculate PE for a body of 10 kg which is resting at a height of 10 m.

Ans: The potential energy of an object with mass $=\mathrm{m} \mathrm{kg}$, at height above the ground $=\mathrm{h}$ $\mathrm{m}$

Gravitational force of attraction on that body $=\mathrm{mg} \mathrm{N}$

To lift that body to $B$ height at $h$ m above the ground.

Force applied to lift this body with a constant velocity $=\mathrm{mg} \mathrm{N}$

Distance moved by the body after applying force = $\mathrm{h} \mathrm{m}$

Work done in lifting the body from a to $B$ distance $=$ Force $\times$ Distance

\mathrm{W}=m g \times h=m g h

Energy cannot be destroyed, hence, this energy is stored as potential energy in the stone.

$ \begin{array}{l} m=10 \mathrm{Kg} \\ g=10 \mathrm{~m} / \mathrm{s}^{2} \\ h=10 \mathrm{~m} \\ \mathrm{P} E=\mathrm{mgh} \\ \mathrm{PE}=m g h=10 \times 10 \times 10=1000 \mathrm{Joules} \end{array} $

15. Prove that the total energy of a ball, having mass m, remains conserved when it is thrown downwards from a height of h.

Ans: According to the law of energy conservation, energy can neither be created nor destroyed, it can only be transformed from one form to another.  

Consider a ball with a mass of m stationary at point A at an elevation h from the ground.

Total energy of ball at position A will be:

Potential energy of the ball = mgh

The body is at rest, hence, its KE = 0

Total energy of ball at position $A=K E+P E=0+m g h=m g h$

Total energy of the ball at ground (position B) will be:

When the body strikes to the ground, its elevation is equal to zero, hence, its potential energy wrt ground $=0$

Velocity of ball when it strikes to the ground (position $\mathrm{B})=$ ?

Its acceleration $=0-\mathrm{g} \mathrm{m} / \mathrm{s}^{2}=-\mathrm{g} \mathrm{m} / \mathrm{s} 2$

Total energy of the body on ground $=K E+P E=m g h+0=m g h$

Total energy of ball at point Cwill be:

Say that, the ball falls through $x$ and be at $C$ during its fall.

Elevation of the body at $\mathrm{C}=\mathrm{h}-\mathrm{x}$

Potential energy at $\mathrm{C}=\mathrm{mg}(\mathrm{h}-\mathrm{x})$

Let the velocity at position $C$ will be $v$

$ \begin{array}{l} v^{2}=u^{2}+2 a s \\ v^{2}=2(-g)(-x)=2 g x \\ K E=\dfrac{1}{2} m v^{2}=\dfrac{1}{2} \times m \times 2 g x=m g x \end{array} $

Total energy at $C=m g(h-x)+m g x=m g h$

It means that the total energy at all points of the fall is always the same.

16. Define power. Prove that the power = force x speed. Can you calculate the power of a 10 kg object accelerating at a speed of 10 m/s 2 and reaching a velocity of 5 m/s?

Ans: Power is the rate of work done.

\text { Power }=\dfrac{\text { Work Done }}{\text { Time taken }}

Unit of Power is watt (w)

$ \begin{array}{l} P=\text { Power }=\dfrac{\text { Work Done }(v)}{\text { ime taken }(t)} \\ P=\dfrac{\text { Force }(F) \times \text { Displacement }(S)}{\text { Time taken }(t)} \\ P=\dfrac{F \times S}{t} \\ \dfrac{S}{t}=v=\text { Velocity } \end{array} $

Mass of the object $(\mathrm{M})=10 \mathrm{Kg}$

Acceleration of the object $(a)=10 \mathrm{~m} / \mathrm{s}^{2}$

And its velocity $(\mathrm{v})=5 \mathrm{~m} / \mathrm{s}$

$ \begin{array}{l} P=F \times v \\ P=F v \\ P=M \times a \times v \\ P=10 \times 10 \times 5 \\ P=500 W \end{array} $

17. What does the unit of electrical energy mean? When the meter displays 400 energy units, how much energy is consumed in joules?

Ans: The unit of electrical energy is defined as the energy consumed (or consumed) by an electrical device with an output power of $1 \mathrm{~kW}$ in one hour.

400 units $=144 \times 10^{7} J$ or 1 unit $=1 k w h$

Now, $1 k w h=1000 w \times 3600 s=3.6 \times 10^{6} w s$

$ \begin{array}{l} 1 w=\dfrac{1 \text { Joule }(J)}{1 \operatorname{second}(S)} \\ 1 \text { unit } \operatorname{mm} K w h=\dfrac{3.6 \times 10^{6} J \times S}{S}=3.6 \times 10^{6} J \end{array} $

So, if 400 units of electrical energy is consumed then,

$ \begin{array}{l} 1 \text { Unit }=3.6 \times 10^{6} J \\ 400 \text { Units }=3.6 \times 10^{6} \times 400 J \\ 400 \text { units }=144 \times 10^{7} J \end{array} $

CBSE Important Questions Class 9 Science Chapter 11

Definition of work.

In simple terms, when any displacement happens, work is said to be done. Work is a form of energy. When a considerable amount of force is applied to someone or something, which carries displacement, upon the subject where force has applied, work is said to be done.

Therefore, two conditions must satisfy for a work to be done:

Applying force is must

Displacement must take place

There can be multiple causes of displacement as per the direction:

(Image to be added soon)

What is Energy?

Energy is typically defined as the capacity to do a certain amount of work. These various kinds typically fall under class 9 work and energy important questions. Let’s discuss in more detail about energy and its distinctive forms.

Which are the Forms of Energy

There are several forms of energy, namely, kinetic, potential, electrical, thermal, nuclear, chemical etc. Let's discuss these in detail.

Kinetic Energy: Our body possesses a certain amount of energy, when in motion. This energy is called kinetic energy. It is always directly proportional to speed. The more is the velocity, the more the work is is required to do to manage that velocity.

Consider an object of mass m moves with uniform velocity u. Then the displacement s will take place because of the constant force F on an object.

We already know the formula for work is, W= F X s

Potential Energy: A body presents a certain amount of energy due to its position or shape. This is called potential energy. It can better be explained through a diagram below

Gravitational Potential: When an object is raised to a specific height h, then the work is said to be done on it against gravity. So such an object is said to acquire GPE.

Therefore, GPE= work done in raising a body from the ground to a point against gravity.

Consider a body of mass m raised through height h, from the ground

The force required to raise the object will be the same as its weight mg.

Derivation:

Object gains energy= work done on it  

Therefore, the work done will be W. This work is done against the gravity.                                                                                      

Attaining same height= same amount of work done, different paths don’t matter.                                                                                                                                                                                                                                                            

Mechanical Energy: kinetic and potential energy together combines to form mechanical energy.

Law of Conservation of Energy

This topic will primarily cover most of the important questions of chapter work and energy class 9. This proves that energy cannot be created or destroyed. It only passes on from one object to another with the same or different form. This can better be explained from a picture.

Also, potential and kinetic energy combine to become constant.

Potential energy happens to be maximum when an object of mass m is bought to height h, ultimately nullifying the kinetic energy. Likewise, the potential energy decreases when the height is decreasing and kinetic energy increase with the increase in v. After that, h will be negligible when the object reaches the ground and velocity will be maximum. So kinetic energy will be equal to or greater than the potential energy. 

Both potential and kinetic energy are inversely proportional. This chapter is exceptionally integral to know about, from an examination perspective. One can also expect to get considerable work and energy class 9 important questions as per previous year analysis. This topic will also be there in the upper classes, so leaving it today can be a hurdle for tomorrow.

Chapter Summary 

CBSE Class 9 Science Chapter 11 - Work and Energy dives into the fascinating world of motion and power. It's all about understanding how things work and why they move. The chapter introduces the concept of energy, which is like the fuel that makes everything happen. We explore the connection between force, work, and energy, unraveling the secrets behind everyday activities. From the basics of what 'work' means in physics to the different forms of energy, it's a journey into the forces that make our world go round.

Benefits of using Vedantu for Class 9 Chapter 11 - Work and Energy

Mastering Class 9 Chapter 11 - Work and Energy becomes seamless with Vedantu's tailored resources. These curated tools encompass NCERT solutions that delve into the core principles of work and energy. Through concise explanations and practice materials, Vedantu elevates understanding, fostering confidence and competence in tackling scientific concepts. Here are the Benefits of using Vedantu for Class 9 Chapter 11 - Work and Energy: 

Focus on key topics for efficient studying.

Prepares students for exams and reduces anxiety.

Reinforces understanding of fundamental concepts.

Teaches effective time management.

Enables self-assessment and progress tracking.

Strategic approach for higher scores.

Covers a wide range of topics for comprehensive understanding.

Supports exam preparation and boosts confidence.

Reviewing all the crucial questions for Class 9 Chapter 11 Science - Work and Energy provides students with a solid grasp of the chapter's topics. The extra and important questions for Class 9 Chapter 11 Science - Work and Energy engage in a concept-focused discussion, encompassing all chapter themes. This question-and-answer method proves time-saving during exam prep, offering an efficient way to revise the chapter and enhance understanding. Practising these important questions streamlines preparation and boosts confidence for the upcoming exams.

Important Related Links for CBSE Class 9 

arrow-right

FAQs on Important Questions for CBSE Class 9 Science Chapter 11 - Work and Energy

1. Which are the most important questions of work and energy class 9?

Ans: It is always suggested to go through the whole chapter diligently. A number of questions are likely to come from the chapter, law of conservation of energy. Other than the important applications, students are also advised to polish their numerical abilities. As, maximum numerical questions often come from this chapter.

2. Which are the best references for understanding physics class 9?

Ans: One can always rely on NCERT for the best references and maximum questions. This book covers a wide range of all the crucial topics from exam perspective. Other than that, Vedantu also offers a great online platform for better understanding of a subject. Student can either delve into the notes and PDF provided on a particular topic or attend a live session with our experts.

3. Are the important questions of chapter work and energy class 9 helpful from future perspective?

Ans: The chapter work and energy is definitely crucial for higher studies. More importantly, one has to make up your mind from here about the stream one is going to select. And if you are planning to go with science for higher studies, you can’t escape the topic. It will cover the wide portion of the higher secondary. So, not just for today, this chapter is essentially important for the future studies.

4. How are work and energy-related in Chapter 11 from Class 9 Science?

Ans: All sorts of work are forms of energy. Work performed on a body is stored as energy. Greater energy is required to accomplish more work. For example, in any kind of work, you do need force, and that force comes from the energy stored in your body.

Chapter 11 of Class 9 Science gives the students various information along with easy to comprehend examples that makes the chapter relatable and fun to learn.  

5. What is work according to Chapter 11 from Class 9 Science?

Ans: When force is applied to displace the object it is known as work. It is calculated with the product of force and displacement and measured in joule. Students should focus on learning the key definitions and formulas from this chapter as they are very scoring. The numerical problems are easy once you practice them on a regular basis. You can effectively retain the chapter if you learn it using real-life examples given in the chapter for references.

6. What is energy according to Chapter 11 of Class 9 Physics?

Ans: Energy is said to be the ability to perform a task. Like work, it is also measured in Joules and the formula for Potential Energy is mgh that is the product of mass, acceleration due to gravity, and height. There are different forms in which energy exists. The students gain more in-depth insight on the various types of energy like kinetic, electromagnetic, and so on in the chapter along with examples for references.

7. Can one create energy?

Ans: It is stated by the Law of conservation of energy that ‘Energy can neither be created nor destroyed’. We can only convert energy from one form to another using different methods. For example, the potential energy of a substance is converted into kinetic energy when we introduce the substance to speed. Solar power is used to produce electricity by saving energy in the solar panels. There are numerous other examples that prove that energy cannot be created but only converted.

8. What are the various types of energy?

Ans: Energy exists in several forms, like kinetic energy, mechanical energy, chemical energy, electrical energy, elastic energy, thermal energy, heat energy, gravitational energy, magnetic energy, nuclear energy, and so on. Some of these energies can be converted in other forms through different man-made processes. For example, kinetic energy is converted into electric energy through hydropower plants. You can get a deeper understanding of these categories of energy in Chapter 11 of Class 9th Science on Vedantu.

CBSE Class 9 Science Important Questions

Cbse study materials.

Free Online study material

  • Neet Online Test Pack

12th Standard stateboard question papers & Study material

தமிழ் subjects.

subject-image

கணினி பயன்பாடுகள்

subject-image

கணினி அறிவியல்

வணிகக் கணிதம் மற்றும் புள்ளியியல்.

subject-image

கணினி தொழில்நுட்பம்

subject-image

கணக்குப்பதிவியல்

subject-image

English Subjects

subject-image

Computer Science

subject-image

Business Maths and Statistics

subject-image

Accountancy

subject-image

Computer Applications

subject-image

Computer Technology

subject-image

11th Standard stateboard question papers & Study material

subject-image

9th Standard stateboard question papers & Study material

subject-image

Social Science

subject-image

சமூக அறிவியல்

6th standard stateboard question papers & study material.

subject-image

10th Standard stateboard question papers & Study material

subject-image

7th Standard stateboard question papers & Study material

subject-image

8th Standard stateboard question papers & Study material

subject-image

கணிதம் - old

subject-image

12th Standard CBSE Subject Question Paper & Study Material

subject-image

Introductory Micro and Macroeconomics

subject-image

Business Studies

subject-image

Indian Society

subject-image

Physical Education

subject-image

Bio Technology

subject-image

Engineering Graphics

subject-image

Entrepreneurship

subject-image

Hindi Elective

subject-image

Home Science

subject-image

Legal Studies

subject-image

Political Science

subject-image

11th Standard CBSE Subject Question Paper & Study Material

subject-image

Mathematics

subject-image

Enterprenership

subject-image

Applied Mathematics

10th standard cbse subject question paper & study material.

subject-image

9th Standard CBSE Subject Question Paper & Study Material

subject-image

8th Standard CBSE Subject Question Paper & Study Material

subject-image

7th Standard CBSE Subject Question Paper & Study Material

subject-image

6th Standard CBSE Subject Question Paper & Study Material

subject-image

School Exams

sub-category-image

Tamil Nadu State Board Exams

sub-category-image

Scholarship Exams

sub-category-image

Study Materials , News and Scholarships

sub-category-image

Stateboard Tamil Nadu

sub-category-image

Free Online Tests

sub-category-image

Educational News

sub-category-image

Scholarships

sub-category-image

Entrance Exams India

sub-category-image

Video Materials

case study based questions class 9 physics work and energy

9th Standard CBSE

Class 9th Science - Work and Energy Case Study Questions and Answers 2022 - 2023

case study based questions class 9 physics work and energy

Class 9th Science - Work and Energy Case Study Questions and Answers 2022 - 2023 Study Materials Sep-09 , 2022

QB365 provides a detailed and simple solution for every Possible Case Study Questions in Class 9th Science Subject - Work and Energy, CBSE. It will help Students to get more practice questions, Students can Practice these question papers in addition to score best marks.

case study based questions class 9 physics work and energy

A PHP Error was encountered

Severity: Warning

Message: in_array() expects parameter 2 to be array, null given

Filename: material/details.php

Line Number: 1436

Message: Use of undefined constant EXAM - assumed 'EXAM' (this will throw an Error in a future version of PHP)

Line Number: 1438

QB365 - Question Bank Software

Work and energy case study questions with answer key.

Final Semester - June 2015

case study based questions class 9 physics work and energy

(ii) What is the value of total energy of the bob at position A ?

(iii) What is the value of kinetic energy of the bob at mean position 'O' ?

(iv) What is the value of kinetic energy and potential energy of the bob at the position 'P' whose height above 'O' is 2 cm ?

(v) What is kinetic energy? (a) Energy acquired due to motion (b) Energy acquired due to rest (c) Sum of Potential and mechanical energy (d) It is the energy stored inside a body.

*****************************************

  • Previous Class 9th Science - Force and Laws of Motion Case Study Questions and Answers 20...
  • Next Class 9th Science - Motion Case Study Questions and Answers 2022 - 2023

Reviews & Comments about Class 9th Science - Work and Energy Case Study Questions and Answers 2022 - 2023

case study based questions class 9 physics work and energy

Write your Comment

case study based questions class 9 physics work and energy

9th Standard CBSE Science Usefull Links

case study based questions class 9 physics work and energy

  • 10th Standard

Other 9th Standard CBSE Subjects

alt-subject-image

Other 9th Standard CBSE Science Study material

Class 9th science - work and energy case study questions and answers 2022 - 2023 click to view, class 9th science - force and laws ... click to view, class 9th science - motion case study questions and answers 2022 - 2023 click to view, class 9th science - the fundamental unit ... click to view, class 9th science - is matter around ... click to view, class 9th science - matter in our ... click to view, 9th standard cbse science annual exam model question 2020 click to view, 9th standard cbse science public exam sample question 2020 click to view, 9th standard cbse science public exam important question 2019-2020 click to view, 9th standard science board exam sample question 2020 click to view, 9th standard science board exam model question 2019-2020 click to view, cbse 9th science - improvement in food resources model question paper click to view, cbse 9th science - nature resources model question paper click to view, cbse 9th science - why do we fall ill model question paper click to view, cbse 9th science - sound model question paper click to view, register & get the solution for class 9th science - work and energy case study questions and answers 2022 - 2023.

  • NCERT Solutions
  • NCERT Class 9
  • NCERT 9 Science
  • Chapter 11: Work and Energy

NCERT Solutions for Class 9 Science Chapter 11: Work and Energy

Ncert solutions class 9 science chapter 11 – cbse free pdf download.

* According to the CBSE Syllabus 2023-24, this chapter has been renumbered as Chapter 10.

NCERT Solutions for Class 9 Science Chapter 11 Work and Energy help you lay a good foundation for your CBSE exam preparation. Students who refer to NCERT Solutions regularly are benefited from the comprehensive methodology and the detailed step-by-step procedure, which will fetch them good marks in their board examinations.

Download Exclusively Curated Chapter Notes for Class 9 Science Chapter – 11 Work and Energy

Download most important questions for class 9 science chapter – 11 work and energy.

NCERT Solutions for Class 9 Science Chapter 11: Work and Energy help students to practise and gain more confidence in their preparations. All the NCERT Class 9 solutions are thoroughly prepared by our team of subject experts, and it includes objectives, diagram-based explanations, and short and long-type questions, which can be useful for both CBSE exams and other competitive exams.

  • Chapter 1 Matter in Our Surroundings
  • Chapter 2 Is Matter Around Us Pure?
  • Chapter 3 Atoms and Molecules
  • Chapter 4 Structure of the Atom
  • Chapter 5 The Fundamental Unit of Life
  • Chapter 6 Tissues
  • Chapter 7 Diversity in Living Organisms
  • Chapter 8 Motion
  • Chapter 9 Force and Laws of Motion
  • Chapter 10 Gravitation
  • Chapter 12 Sound
  • Chapter 13 Why Do We Fall ill?
  • Chapter 14 Natural Resources
  • Chapter 15 Improvement in Food Resources

carouselExampleControls111

ncert solutions for class 9 march 29 science chapter 11 work and energy 01

Previous Next

BYJUS Classes 9

Access Answers of Science NCERT Class 9 Chapter 11: Work and Energy (All intext and exercise questions solved)

Exercise-11.1 Page: 148

1. A force of 7 N acts on an object. The displacement is, say 8 m, in the direction of the force. Let us take it that the force acts on the object through the displacement. What is the work done in this case?

When a force F acts on an object to move it in its direction through a distance S,  work is done.

The work on the body is done by force.

Work done = Force × Displacement

F = 7 N S = 8 m

So, work done,

Exercise-11.2 Page: 149

1. When do we say that work is done?

Work is completed whenever the given conditions are satisfied:

(i) A force acts on the body.

(ii) There’s a displacement of the body by applying force in or opposite to the direction of the force.

2. Write an expression for the work done when a force is acting on an object in the direction of its displacement.

When a force F displaces a body through a distance S within the direction of the applied force, then the work done W on the body is given by the expression:

3. Define 1 J of work.

1 J is the amount of work done on an object when a force of 1 N displaces it by 1 m along the line of action of the force.

4. A pair of bullocks exerts a force of 140 N on a plough. The field being ploughed is 15 m long.

How much work is done in ploughing the length of the field?

Work done by the bullocks is given by the expression:

Applied force, F = 140 N

Displacement, d = 15 m

W = 140 x 15 = 2100 J

Therefore, 2100 J of work is done in ploughing the length of the field.

Exercise-11.3 Page: 152

1. What is the kinetic energy of an object?

The energy possessed by a body by virtue of its motion is termed mechanical energy or kinetic energy. Every moving object possesses mechanical energy. A body uses mechanical energy to try to work. The kinetic energy of the hammer is employed in driving a nail into a log of wood, the mechanical energy of air is employed to run wind mills, etc.

2. Write an expression for the kinetic energy of an object.

If a body of mass m is moving with a speed v, then its K.E. E k is given by the expression,

E k = 1/2 m v 2

Its SI unit is Joule (J).

3. The kinetic energy of an object of mass, m moving with a velocity of 5 ms -1 is 25 J. What will be its kinetic energy when its velocity is doubled? What will be its kinetic energy when its velocity is increased three times?

K.E. of the object = 25J

Velocity of the object (v) = 5 m/s

K.E. = (½) mv 2

25 = (½) m (5) 2

50 = 25 x m

Now, when velocity is doubled

K.E. = (½) x 2 x (10) 2

K.E. = 10 2

K.E. = 100 J

When velocity is increased three times, then

K.E. = (½) x 2 x (15) 2

K.E. = (15) 2

K.E. = 225 J

Exercise-11.4 Page: 156

1. What is power?

Power is defined as the rate of doing work or the rate of transfer of energy. If an agent does a work W in time t, then power is given by: 

NCERT Solutions for Class 9 Science - Chapter 11 Image 2

It is expressed in watt (W).

2. Define 1 watt of power.

A body is claimed to possess power of one watt if it works at the speed of 1 joule in 1 s.

One W = 1 J/1 S

3. A lamp consumes 1000 J of electrical energy in 10 s. What is its power?

Power = Work/Time

Time = 10 s

Work done = Energy consumed by the lamp = 1000 J

Power = 1000/10 = 100 Js -1 =100 W

Hence, the power of the lamp is 100 W

4. Define average power.

Average power is defined as the ratio of total work done by the body to the total time taken by the body.

NCERT Solutions for Class 9 Science - Chapter 11 Image 3

Exercises – 11.5 Page: 158

1. Look at the activities listed below. Reason out whether or not work is done in the light of your understanding of the term ‘work’.

(a) Suma is swimming in a pond.

(b)A donkey is carrying a load on its back.

(c) A wind-mill is lifting water from a well.

(d) A green plant is carrying out photosynthesis.

(e) An engine is pulling a train.

(f) Food grains are getting dried in the sun.

(g) A sailboat is moving due to wind energy.

Work is finished whenever the given 2 conditions are satisfied:

(a) While swimming, Suma applies a force to push the water backwards. Therefore, Suma swims in the forward direction caused by the forward reaction of water. Here, the force causes a displacement. Hence, the work is done by Seema while swimming.

(b) While carrying a load, the donkey has to apply a force in the upward direction. But, displacement of the load is in the forward direction. Since displacement is perpendicular to force, the work done is zero.

(c) A windmill works against gravity to elevate water. The windmill lift water by applying a force in an upward direction, and thus the water is moving in the same upward direction itself. Hence, work is done by the windmill to lift water from the well.

(d) No force is required when a green plant is carrying out photosynthesis. The plant does not exert any force to move. Since there is no displacement or force. Hence, no work is done.

(e) When an engine is pulling a train, it is applying a force in the forward direction. So, it is moving in the forward direction. Since displacement and force are in the same direction. Hence, work is done by the engine.

(f) There is no force involved in the process of drying food grains in the sun and the grains do not move. Since there is no force or displacement. Hence, no work is done.

(g) When a sailboat is moving due to wind energy, it is applying force in the forward direction. So, it is moving in the forward direction. Since displacement and force are in the same direction. Hence, work is done. 

2. An object thrown at a certain angle to the ground moves in a curved path and falls back to the ground. The initial and the final points of the path of the object lie on the same horizontal line. What is the work done by the force of gravity on the object?

Work done by the force of gravity on an object depends solely on vertical displacement. Vertical displacement is given by the distinction in the initial and final positions/heights of the object which is zero.

Work done by gravity is given by the expression, 

W = m × g × h

h= Vertical displacement = 0

W = m g × 0 = 0 J

Hence, the work done by the gravity on the given object is zero joule.

3. A battery lights a bulb. Describe the energy changes involved in the process.

When a bulb is connected to a battery, then the energy of the battery is transferred into voltage. Once the bulb receives this voltage, then it converts it into light and heat energy. Hence, the transformation of energy in the given situation can be shown as:

Chemical Energy → Electrical Energy → Light Energy + Heat Energy.

4. Certain force acting on a 20 kg mass changes its velocity from 5 m s -1 to 2 m s -1 . Calculate the work done by the force.

Initial velocity u = 5 m/s

Mass of the body = 20kg

Final velocity v = 2 m/s

The initial kinetic energy

E i = (1/2) mu 2 = (1/2) × 20 × (5) 2 

Final kinetic energy 

E f = (1/2) mv 2 = (1/2) × 20 × (2) 2  

Work done = Change in kinetic energy

Work done = E f – E i

Work done = 40 J – 250 J

Work done = -210 J

5. A mass of 10 kg is at a point A on a table. It is moved to a point B. If the line joining A and B is horizontal, what is the work done on the object by the gravitational force? Explain your answer.

Work done by gravity depends solely on the vertical displacement of the body. It doesn’t rely on the trail of the body. Therefore, work done by gravity is given by the expression,

Vertical displacement, h = 0

∴ W= m × g × zero = 0

Therefore the work done on the object by gravity is zero.

6. The potential energy of a freely falling object decreases progressively. Does this violate the law of conservation of energy? Why?

No, the method doesn’t violate the law of conservation of energy. This is because when the body falls from a height, its potential energy changes into kinetic energy progressively. A decrease in the potential energy is equal to an increase in the kinetic energy of the body. Throughout the method, the total mechanical energy of the body remains conserved. Therefore, the law of conservation of energy isn’t desecrated.

7. What are the various energy transformations that occur when you are riding a bicycle?

During riding a bicycle, the muscular energy of the rider regenerates into heat and mechanical energy. Kinetic energy provides a rate to the bicycle, and warmth energy heats our body.

Muscular energy → mechanical energy + heat

8. Does the transfer of energy take place when you push a huge rock with all your might and fail to move it? Where is the energy you spend going?

When we push a huge rock, there’s no transfer of muscular energy to the stationary rock. Also, there’s no loss of energy since muscular energy is transferred into heat energy, which causes our body to become hot.

9. A certain household has consumed 250 units of energy during a month. How much energy is this in joules?

1 unit of energy = 1kWh

Energy (E) = 250 units

1 unit = 1 kWh

1 kWh = 3.6 x 10 6 J

Therefore, 250 units of energy = 250 × 3.6 × 10 6

= 9 × 10 8 J.

10. An object of mass 40 kg is raised to a height of 5 m above the ground. What is its potential energy? If the object is allowed to fall, find its kinetic energy when it is half-way down.

Given Mass (m) = 40 kg

Acceleration due to gravity (g)= 10m/s²

Height (h)= 5m

Potential energy= m × g× h

P.E= 40 × 10 × 5 = 2000J

Potential energy = 2000J ( 2000 joules)

At a height of 5 metres, the object has a potential energy of 2000 J.

When this object is allowed to fall and it is halfway down, its height above the ground will be half of 5 m= 5/2= 2.5m.

P.E at Halfway down= m× g×h

P.E= 40× 10 × 2.5= 1000J

Potential Energy halfway down= 1000 joules.

According to the law of conservation of energy:

Total potential energy= potential energy halfway down+ kinetic energy halfway down

2000 = 1000 + K.E halfway down

K.E at halfway down= 2000- 1000= 1000 J

Kinetic energy at halfway down= 1000 joules .

case study based questions class 9 physics work and energy

11. What is the work done by the force of gravity on a satellite moving round the earth? Justify your answer.

Work is completed whenever the given two conditions are satisfied:

If the direction of force is perpendicular to displacement, then the work done is zero. When a satellite moves around the Earth, then the force of gravity on the satellite is perpendicular to its displacement. Therefore, the work done on the satellite by the Earth is zero.

12. Can there be displacement of an object in the absence of any force acting on it? Think. Discuss this question with your friends and teacher

Yes, there can be displacement of an object in the absence of any force acting on it.  If a single force acts on an object, the object accelerates. If an object accelerates, a force is acting on it.

Assume an object is moving with constant velocity. The net force acting on it is zero. But, there is a displacement along with the motion of the object. Therefore, there can be a displacement without a force.

13. A person holds a bundle of hay over his head for 30 minutes and gets tired. Has he done some work or not? Justify your answer.

Work is completed whenever the given 2 conditions are satisfied.

When an individual holds a bundle of hay over his head, there is no displacement in the hay bundle. Although the force of gravity is acting on the bundle, the person isn’t applying any force on it. Therefore, in the absence of force, work done by the person on the bundle is zero. 

14. An electric heater is rated 1500 W. How much energy does it use in 10 hours?

Power of the heater = 1500 W = 1.5 kW

Time taken = 10 hours

Energy consumed by an electric heater can be obtained with the help of the expression,

Power = Energy consumed / Time taken

Energy consumed = Power x Time taken

Energy consumed = 1.5 x 10

Energy consumed = 15 kWh

Therefore, the energy consumed by the heater in 10 hours is 15 kWh.

15. Illustrate the law of conservation of energy by discussing the energy changes which occur when we draw a pendulum bob to one side and allow it to oscillate. Why does the bob eventually come to rest? What happens to its energy eventually? Is it a violation of the law of conservation of energy?

Consider the case of an oscillation pendulum.

NCERT Solutions for Class 9 Science - Chapter 11 Image 4

When an apparatus moves from its mean position P to either of its extreme positions A or B, it rises through a height h on top of the mean level P. At this time, the K.E. of the bob changes fully into P.E. The K.E. becomes zero, and also, the bob possesses P.E. solely. Because it moves towards purpose P, its P.E. decreases increasingly. Consequently, the K.E. will increase. Because the bob reaches purpose P, its P.E. becomes zero, and also, the bob possesses K.E. solely. This method is perennial as long as the apparatus oscillates.

The bob doesn’t oscillate forever. It involves rest as a result of air resistance resisting its motion. The apparatus loses its K.E. to beat this friction and stops once in a while. The law of conservation of energy isn’t desecrated because the energy lost by the apparatus to beat friction is gained by its surroundings. Hence, the overall energy of the apparatus and also the encompassing system stay preserved.

16. An object of mass, m is moving with a constant velocity, v. How much work should be done on the object in order to bring the object to rest?

The kinetic energy of an object of mass m, moving with a velocity, v, is given by the expression,

K.E = 1/2 mv 2

In order to bring it to rest, its velocity has to be reduced to zero, and in order to accomplish that, the kinetic energy has to be drained off and sent somewhere else.

An external force has to absorb energy from the object, i.e. do negative work on it, equal to its kinetic energy, or

– 1/2 mv 2 .

17. Calculate the work required to be done to stop a car of 1500 kg moving at a velocity of 60 km/h.

Given data:

The mass of the body = 1500kg

Velocity v = 60km/hr

NCERT Solutions for Class 9 Science - Chapter 11 Image 7

The work required to stop the moving car = change in kinetic energy

18. In each of the following a force, F is acting on an object of mass, m. The direction of displacement is from west to east shown by the longer arrow. Observe the diagrams carefully and state whether the work done by the force is negative, positive or zero.

NCERT Solutions for Class 9 Science - Chapter 11 Image 9

In this case, the direction of force functioning on the block is perpendicular to the displacement. Therefore, work done by force on the block will be zero.

In this case, the direction of force functioning on the block is in the direction of displacement. Therefore, work done by force on the block will be positive.

In this case, the direction of force functioning on the block is contrary to the direction of displacement. Therefore, work done by force on the block will be negative.

19. Soni says that the acceleration in an object could be zero even when several forces are acting on it. Do you agree with her? Why?

Acceleration in an object could be zero even when many forces work on it. This happens when all the forces get rid of one another, i.e., the online force working on the object is zero. For a uniformly moving object, the online force working on the it is zero. Hence, the acceleration of the thing is zero. Hence, Soni is correct.

20. Find the energy in kW h consumed in 10 hours by four devices of power 500 W each.

Power rating of the device (P) = 500 W = 0.50 kW

Time for which the device runs (T) = 10 h

Energy consumed by an electric device can be obtained by the expression

Power = Energy consumed/Time taken

∴ Energy consumed = Power × Time

Energy consumed = 0.50 × 10

Energy consumed = 5 kWh

Thus, the energy consumed by four equal rating devices in 10 h will be

⇒ 4 × 5 kWh

21. A freely falling object eventually stops on reaching the ground. What happens to its kinetic energy?

When an object falls freely towards the ground, its potential energy decreases, and kinetic energy increases; as the object touches the ground, all its potential energy becomes kinetic energy. Since the object hits the ground, all its kinetic energy becomes heat energy and sound energy. It can also deform the ground depending upon the ground’s nature and the amount of kinetic energy possessed by the object.

Work and Energy is one of the important topics in the Class 9 Science curriculum, and the expected weightage is 27. Every student should practise these NCERT Solutions as there are more solved numericals which are repetitively asked in the exams. Apart from the solved examples, these solutions also include key notes and important terminologies from the exam point of view.

Topic covered in NCERT Solutions

  • Work – 5 Questions (3 long, 2 short)
  • Power -5 Questions (1 long, 4 short)
  • Energy 1 Question (1 short)
  • Unit of Energy 1 Question (1 MCQ)
  • Forms of Energy 1 Question (1 MCQ)
  • Kinetic Energy and its expression 1 Question (1 short)
  • Potential Energy and its expression 1 Question (1 short)
  • Conservation of Energy 1 Question (1 short)

Class 9 Chapter 11: Work and Energy is an important topic that provides a foundation for all your future studies. Work and Energy are closely related terms which are quite often used in our daily lives. Here, in this topic, students learn more in detail about work, power, force, energy, and how they are interrelated to each other. NCERT Solutions for Class 9 Science Chapter 11: Work and Energy include more solved problems and other daily basis examples, which help students to learn the topic effectively.

Key Features of NCERT Solutions for Class 9 Science Chapter 11: Work and Energy

  • Provides detailed explanations for all complex topics.
  • Provides completely solved solutions to all the questions in the Class 9 Science NCERT textbooks.
  • These solutions include important questions at the end of every chapter
  • The language used in NCERT Solutions for Class 9 Science is easy and simple to understand for the students.
  • All these solutions are prepared by the subject experts after extensive research on every topic in order to provide appropriate and genuine information to the students.

Disclaimer:

Dropped Topics –  11.3.1 Commercial Unit of Energy.

Frequently Asked Questions on NCERT Solutions for Class 9 Science Chapter 11

What are the topics covered under chapter 11 of ncert solutions for class 9 science, what is the kinetic energy of an object, as given in chapter 11 of ncert solutions for class 9 science, can students rely on the ncert solutions for class 9 science chapter 11 from byju’s, leave a comment cancel reply.

Your Mobile number and Email id will not be published. Required fields are marked *

Request OTP on Voice Call

Post My Comment

case study based questions class 9 physics work and energy

Good Solutions very nice keep it up 👍👍🤓👌👌

case study based questions class 9 physics work and energy

  • Share Share

Register with BYJU'S & Download Free PDFs

Register with byju's & watch live videos.

close

Trial Class

myCBSEguide

  • Class 9 Science Case...

Class 9 Science Case Study Questions

Table of Contents

myCBSEguide App

Download the app to get CBSE Sample Papers 2023-24, NCERT Solutions (Revised), Most Important Questions, Previous Year Question Bank, Mock Tests, and Detailed Notes.

If you are wondering how to solve class 9 science case study questions, then myCBSEguide is the best platform to choose. With the help of our well-trained and experienced faculty, we provide solved examples and detailed explanations for the recently added Class 9 Science case study questions.

You can find a wide range of solved case studies on myCBSEguide, covering various topics and concepts. Class 9 Science case studies are designed to help you understand the application of various concepts in real-life situations.

The rationale behind Science

Science is crucial for Class 9 students’ cognitive, emotional, and psychomotor development. It encourages curiosity, inventiveness, objectivity, and aesthetic sense.

In the upper primary stage, students should be given a variety of opportunities to engage with scientific processes such as observing, recording observations, drawing, tabulating, plotting graphs, and so on, whereas in the secondary stage, abstraction and quantitative reasoning should take a more prominent role in science teaching and learning. As a result, the concept of atoms and molecules as matter’s building units, as well as Newton’s law of gravitation, emerges.

Science is important because it allows Class 9 Science students to understand the world around us. It helps to find out how things work and to find solutions to problems at the Class 9 Science level. Science is also a source of enjoyment for many people. It can be a hobby, a career, or a source of intellectual stimulation.

Case study questions in Class 9 Science

The inclusion of case study questions in Class 9 science CBSE is a great way to engage students in critical thinking and problem-solving. By working through real-world scenarios, Class 9 Science students will be better prepared to tackle challenges they may face in their future studies and careers. Class 9 Science Case study questions also promote higher-order thinking skills, such as analysis and synthesis. In addition, case study questions can help to foster creativity and innovation in students. As per the recent pattern of the Class 9 Science examination, a few questions based on case studies/passages will be included in the CBSE Class 9 Science Paper. There will be a paragraph presented, followed by questions based on it.

Examples of Class 9 science class case study questions

Class 9 science case study questions have been prepared by myCBSEguide’s qualified teachers. Class 9 case study questions are meant to evaluate students’ knowledge and comprehension of the material. They are not intended to be difficult, but they will require you to think critically about the material. We hope you find Class 9 science case study questions beneficial and that they assist you in your exam preparation.

The following are a few examples of Class 9 science case study questions.

Class 9 science case study question 1

  • due to its high compressibility
  • large volumes of a gas can be compressed into a small cylinder
  • transported easily
  • all of these
  • shape, volume
  • volume, shape
  • shape, size
  • size, shape
  • the presence of dissolved carbon dioxide in water
  • the presence of dissolved oxygen in the water
  • the presence of dissolved Nitrogen in the water
  • liquid particles move freely
  • liquid have greater space between each other
  • both (a) and (b)
  • none of these
  • Only gases behave like fluids
  • Gases and solids behave like fluids
  • Gases and liquids behave like fluids
  • Only liquids are fluids

Answer Key:

  • (d) all of these
  • (a) shape, volume
  • (b) the presence of dissolved oxygen in the water
  • (c) both (a) and (b)
  • (c) Gases and liquids behave like fluids

Class 9 science case study question 2

  • 12/32 times
  • 18 g of O 2
  • 18 g of CO 2
  • 18 g of CH 4
  • 1 g of CO 2
  • 1 g of CH 4 CH 4
  • 2 moles of H2O
  • 20 moles of water
  • 6.022  ×  1023 molecules of water
  • 1.2044  ×  1025 molecules of water
  • (I) and (IV)
  • (II) and (III)
  • (II) and (IV)
  • Sulphate molecule
  • Ozone molecule
  • Phosphorus molecule
  • Methane molecule
  • (c) 8/3 times
  • (d) 18g of CH ​​​​​4
  • (c) 1g of H ​​​​​​2
  • (d) (II) and (IV)
  • (c) phosphorus molecule

Class 9 science case study question 3

  • collenchyma
  • chlorenchyma
  • It performs photosynthesis
  • It helps the aquatic plant to float
  • It provides mechanical support
  • Sclerenchyma
  • Collenchyma
  • Epithelial tissue
  • Parenchyma tissues have intercellular spaces.
  • Collenchymatous tissues are irregularly thickened at corners.
  • Apical and intercalary meristems are permanent tissues.
  • Meristematic tissues, in its early stage, lack vacuoles, muscles
  • (I) and (II)
  • (III) and (I)
  • Transpiration
  • Provides mechanical support
  • Provides strength to the plant parts
  • None of these
  • (a) Collenchyma
  • (b) help aquatic plant to float
  • (b) Sclerenchyma
  • (d) Only (III)
  • (c) provide strength to plant parts

Cracking Class 9 Science Case Study Questions

There is no one definitive answer to Class 9 Science case study questions. Every case study is unique and will necessitate a unique strategy. There are, nevertheless, certain general guidelines to follow while answering case study questions.

  • To begin, double-check that you understand the Class 9 science case study questions. Make sure you understand what is being asked by reading it carefully. If you’re unclear, seek clarification from your teacher or tutor.
  • It’s critical to read the Class 9 Science case study material thoroughly once you’ve grasped the question. This will provide you with a thorough understanding of the problem as well as the various potential solutions.
  • Brainstorming potential solutions with classmates or other students might also be beneficial. This might provide you with multiple viewpoints on the situation and assist you in determining the best solution.
  • Finally, make sure your answer is presented simply and concisely. Make sure you clarify your rationale and back up your claim with evidence.

A look at the Class 9 Science Syllabus

The CBSE class 9 science syllabus provides a strong foundation for students who want to pursue a career in science. The topics are chosen in such a way that they build on the concepts learned in the previous classes and provide a strong foundation for further studies in science. The table below lists the topics covered in the Class 9 Science syllabus of the Central Board of Secondary Education (CBSE). As can be seen, the Class 9 science syllabus is divided into three sections: Physics, Chemistry and Biology. Each section contains a number of topics that Class 9 science students must study during the course.

CBSE Class 9 Science (Code No. 086)

Theme: Materials Unit I: Matter-Nature and Behaviour Definition of matter; solid, liquid and gas; characteristics – shape, volume, density; change of state-melting (absorption of heat), freezing, evaporation (cooling by evaporation), condensation, sublimation. Nature of matter:  Elements, compounds and mixtures. Heterogeneous and homogenous mixtures, colloids and suspensions. Particle nature and their basic units:  Atoms and molecules, Law of constant proportions, Atomic and molecular masses. Mole concept: Relationship of mole to mass of the particles and numbers. Structure of atoms:  Electrons, protons and neutrons, valency, the chemical formula of common compounds. Isotopes and Isobars.

Theme: The World of the Living Unit II: Organization in the Living World Cell – Basic Unit of life:  Cell as a basic unit of life; prokaryotic and eukaryotic cells, multicellular organisms; cell membrane and cell wall, cell organelles and cell inclusions; chloroplast, mitochondria, vacuoles, endoplasmic reticulum, Golgi apparatus; nucleus, chromosomes – basic structure, number. Tissues, Organs, Organ System, Organism: Structure and functions of animal and plant tissues (only four types of tissues in animals; Meristematic and Permanent tissues in plants).

Theme: Moving Things, People and Ideas Unit III: Motion, Force and Work Motion:  Distance and displacement, velocity; uniform and non-uniform motion along a straight line; acceleration, distance-time and velocity-time graphs for uniform motion and uniformly accelerated motion, derivation of equations of motion by graphical method; elementary idea of uniform circular motion. Force and Newton’s laws:  Force and Motion, Newton’s Laws of Motion, Action and Reaction forces, Inertia of a body, Inertia and mass, Momentum, Force and Acceleration. Elementary idea of conservation of Momentum. Gravitation:  Gravitation; Universal Law of Gravitation, Force of Gravitation of the earth (gravity), Acceleration due to Gravity; Mass and Weight; Free fall. Floatation:  Thrust and Pressure. Archimedes’ Principle; Buoyancy. Work, energy and power:  Work done by a Force, Energy, power; Kinetic and Potential energy; Law of conservation of energy. Sound:  Nature of sound and its propagation in various media, speed of sound, range of hearing in humans; ultrasound; reflection of sound; echo.

Theme: Food Unit IV: Food Production Plant and animal breeding and selection for quality improvement and management; Use of fertilizers and manures; Protection from pests and diseases; Organic farming.

PRESCRIBED BOOKS:

  • Science-Textbook for class IX-NCERT Publication
  • Assessment of Practical Skills in Science-Class IX – CBSE Publication
  • Laboratory Manual-Science-Class IX, NCERT Publication
  • Exemplar Problems Class IX – NCERT Publication

myCBSEguide: A true helper

There are numerous advantages to using myCBSEguide to achieve the highest results in Class 9 Science.

  • myCBSEguide offers high-quality study materials that cover all of the topics in the Class 9 Science curriculum.
  • myCBSEguide provides practice questions and mock examinations to assist students in the best possible preparation for their exams.
  • On our myCBSEguide app, you’ll find a variety of solved Class 9 Science case study questions covering a variety of topics and concepts. These case studies are intended to help you understand how certain principles are applied in real-world settings
  • myCBSEguide is that the study material and practice problems are developed by a team of specialists who are always accessible to assist students with any questions they may have. As a result, students may be confident that they will receive the finest possible assistance and support when studying for their exams.

So, if you’re seeking the most effective strategy to study for your Class 9 Science examinations, myCBSEguide is the place to go!

Test Generator

Create question paper PDF and online tests with your own name & logo in minutes.

Question Bank, Mock Tests, Exam Papers, NCERT Solutions, Sample Papers, Notes

Related Posts

  • Competency Based Learning in CBSE Schools
  • Class 11 Physical Education Case Study Questions
  • Class 11 Sociology Case Study Questions
  • Class 12 Applied Mathematics Case Study Questions
  • Class 11 Applied Mathematics Case Study Questions
  • Class 11 Mathematics Case Study Questions
  • Class 11 Biology Case Study Questions
  • Class 12 Physical Education Case Study Questions

Leave a Comment

Save my name, email, and website in this browser for the next time I comment.

If you're seeing this message, it means we're having trouble loading external resources on our website.

If you're behind a web filter, please make sure that the domains *.kastatic.org and *.kasandbox.org are unblocked.

To log in and use all the features of Khan Academy, please enable JavaScript in your browser.

Unit 4: Work & Energy

About this unit.

In this chapter, we will define the terms 'work' and 'energy'. We will learn how to calculate them, and use them to look at our world in a very different way.

  • Intro to work (Opens a modal)
  • Positive & negative work (Opens a modal)
  • Work done on lifting/falling things - Solved numerical (Opens a modal)
  • Energy intro (kinetic & potential) (Opens a modal)
  • Kinetic energy derivation (Opens a modal)
  • Gravitational potential energy derivation (Opens a modal)
  • Work done by gravity (path independent) (Opens a modal)
  • Using the kinetic energy equation 4 questions Practice

Work energy theorem

  • Work-energy theorem (Opens a modal)
  • Work done from kinetic energy - solved example (Opens a modal)
  • Calculating change in kinetic energy from a force 4 questions Practice

Law of conservation of energy

  • Law of energy conservation (Opens a modal)
  • Energy conservation - solved example (Opens a modal)
  • Power (Opens a modal)
  • Relating power and energy 4 questions Practice

Commercial unit of energy

  • Commercial unit of electrical energy (Opens a modal)
  • Solved example - Cost of operation of electrical device (Opens a modal)

CBSE Expert

Case Study Questions of Chapter 11 Work and Energy PDF Download

Case study Questions on Class 9 Science Chapter 11  are very important to solve for your exam. Class 9 Science Chapter 11 Case Study Questions have been prepared for the latest exam pattern. You can check your knowledge by solving case study-based questions for Class 9 Science Chapter 11 Work and Energy

case study based questions class 9 physics work and energy

In CBSE Class 9 Science Paper, Students will have to answer some questions based on Assertion and Reason. There will be a few questions based on case studies and passage-based as well. In that, a paragraph will be given, and then the MCQ questions based on it will be asked.

Work and Energy Case Study Questions With answers

Here, we have provided case-based/passage-based questions for Class 9 Science  Chapter 11 Work and Energy

Case Study/Passage Based Questions

Figure shows a watch glass embedded in clay. A tiny spherical ball is placed at the edge B at a height h above the centre A

case study based questions class 9 physics work and energy

The kinetic energy of the ball, when it reaches at point A is (a) zero (b) maximum (c) minimum (d) can’t say

Answer: (b) maximum

The ball comes to rest because of (a) frictional force (b) gravitational force (c) both (a) and (b) (d) none of these

Answer: (c) both (a) and (b)

The energy possessed by ball at point C is (a) potential energy (b) kinetic energy (c) both potential and kinetic energy (d) heat energy.

Answer: (a) potential energy

The principle of conservation of energy states that the energy in a system can neither be created nor be destroyed. It can only be transformed from one form to another, but total energy of the system remains constant. Conservation of electrical energy to various forms or vice versa along with devices is illustrated in the figure given below.

case study based questions class 9 physics work and energy

Water stored in a dam possesses (a) no energy (b) electrical energy (c) kinetic energy (d) potential energy.

Answer: (d) potential energy.

A battery lights a bulb. Describe the energy changes involved in the process. (a) Chemical energy →Light energy → Electrical energy (b) Electrical energy → Chemical energy → Electrical energy (c) Chemical energy → Electrical energy → Light energy (d) None of these.

Answer: (c) Chemical energy → Electrical energy → Light energy

Name a machine that transforms muscular energy into useful mechanical work. (a) Amicrophone (b) Bicycle (c) Electric torch (d) An electric bell

Answer: (b) Bicycle

A body is falling from a height of h. After it has fallen a height h/2 , it will possess (a) only potential energy (b) only kinetic energy (c) half potential and half kinetic energy (d) more kinetic and less potential energy.

Answer: (c) half potential and half kinetic energy

An elevator weighing 500 kg is to be lifted up at a constant velocity of 0.4 m s –1 . For this purpose a motor of required horse power is used

The power of motor is (a) 1940 W (b) 1950 W (c) 1960 W (d) 1970 W

Answer: (c) 1960 W

The power of motor in hp is (a) 2.33 (b) 2.43 (c) 2.53 (d) 2.63

Answer: (d) 2.63

Hope the information shed above regarding Case Study and Passage Based Questions for Class 9 Science Chapter 11 Work and Energy with Answers Pdf free download has been useful to an extent. If you have any other queries of CBSE Class 9 Science Work and Energy Case Study and Passage Based Questions with Answers, feel free to comment below so that we can revert back to us at the earliest possible

Leave a Comment Cancel reply

Save my name, email, and website in this browser for the next time I comment.

Download India's best Exam Preparation App Now.

Key Features

  • Revision Notes
  • Important Questions
  • Previous Years Questions
  • Case-Based Questions
  • Assertion and Reason Questions

No thanks, I’m not interested!

case study based questions class 9 physics work and energy

Important Questions for Class 9 Science Chapter 11 Work and Energy

Get important questions for Class 9 Science Chapter 11 Work and Energy with PDF. Our subject expert prepared these  important questions  and answers as per the latest NCERT textbook. These important questions will be helpful to revise the important topics and concepts. You can easily download all the questions and answers in PDF format from our app.

Work and Energy Class 9 Science Important Questions with Answers

Important Questions for Class 9 Science Chapter 11 Work Power and Energy 00001

Gurukul of Excellence

Gurukul of Excellence

Classes for Physics, Chemistry and Mathematics by IITians

Join our Telegram Channel for Free PDF Download

Work & Energy

  • Last modified on: 3 years ago
  • Reading Time: 27 Minutes

01

For doing work, energy is required. • In animals, energy is supplied by food they eat. • In machine, energy is supplied by fuel.

Not much work inspite of working hard : Reading, writing, drawing, thinking, analysing are all energy consuming. But in scientific manner, no work is done in above cases.

       • Example :

• A man is completely exhausted in trying to push a rock (wall), but work done is zero as wall is stationary.

• A man standing still with heavy suitcase may be tired soon but he does no work in this situation as he is stationary.

3.JPG

Work is said to be done when : (i) a moving object comes to rest. (ii) an object at rest starts moving. (iii) velocity of an object changes. (iv) shape of an object changes.

Scientific Conception of Work • Work is done when a force produces motion in a body. • Work is said to be done when a force is applied on a body and the body moves under the influence of force.

Condition of Work (i) Force should be applied on the body. (ii) Body should be displaced.

4.JPG

Work is done when : (i) A cyclist is pedaling the cycle. (ii) A man is lifting load in upward or downward direction.

Work is not done when : (i) A coolie carrying some load on his head stands stationary. (ii) A man is applying force on a big rock.

Work Done by a Fixed Force Work done in moving a body is equal to the product of force and displacement of body in the direction of force.

5.JPG

Work = Force × Displacement W = F × S Work is a scalar quantity.

Unit of Work

Unit of work is Newton metre or Joule.

When a force of 1 Newton moves a body through a distance of 1 metre in its own direction, then the work done is 1 Joule. 1 Joule = 1 Newton × 1 metre 1 J = 1 Nm

6.JPG

The amount of work done depends on the following factors :

(i) Magnitude of force : Greater the force, greater is the amount of work & vice versa.

(ii) Displacement : Greater the displacement, greater is the amount of work & vice-versa.

Negative, Positive and Zero Work

Work done by a force can be positive, negative or zero. (i) Work done is positive when a force acts in the direction of motion of the body.[ Fig. (a)] (θ = 0º).

          Example :

A child pulls a toy car with a string horizontally on the ground.

Here work done is positive. W = F × S

7.JPG

(ii) Work done is negative when a force acts opposite to the direction of motion of the body.

Example : When we kick a football lying on the ground, the force of our kick moves the football. Here direction of force applied & motion of football is same so work done is positive. But when football slows due to force of friction acting in a direction opposite to direction of motion of football [Fig. (b)], work done is negative.

8.JPG

(iii) Work done is zero when a force acts at right angles to the direction of motion.

The moon moves around the earth in circular path. Here force of gravitation acts on the moon at right angles to the direction of motion of the moon. So work done is zero.

9.JPG

•    -ve (negative) sign indicates that work is done against gravity.

Note that if work is done against the direction of motion (gravity), then it is taken –ve.

Example. A coolie lifts a luggage of 15 kg from the ground and put it on his head 1.5 m above the ground. Calculate the work done by him on the luggage.

Solution : Mass of luggage (m) = 15 kg Displacement (S) = 1.5 m So, Work done, W = F × S         [F = mg] = 15 × 10 × 1.5 [g = 10 m/s^2]  [g = force of gravity] = 225.0 kg m/s^2 = 225 N-m = 225 J Hence, work done = 225 J.

(i) The sun is the biggest source of energy. (ii) Most of the energy sources are derived from the sun. (iii) Some energy is received from nucleus of atoms, interior of the earth and the tides.

Definition : The capacity of doing work is known as energy.

The amount of energy possessed by a body is equal to the amount of work it can do. Working body losses energy, body on which work is done gains energy.

Energy is a scalar quantity.

Unit : The SI unit of energy is Joule (J) and its bigger unit is kilo joule (kJ). 1 kJ = 1000 J The energy required to do 1 Joule of work is called 1 Joule energy.

Forms of Energy

Main forms of energy are :

(i) Kinetic energy

(ii) Potential energy

(iii) Heat energy

(iv) Chemical energy

(v) Electrical energy

(vi) Light energy

(vii) Sound energy

(viii) Nuclear energy

• Sum of kinetic energy & potential energy of a body is called mechanical energy.

Mechanical energy

The energy possessed by a body on account of its motion or position is called mechanical energy.

Kinetic Energy

The energy of a body due to its motion is called kinetic energy.

Examples of kinetic energy :

• A moving cricket ball • Running water • A moving bullet • Flowing wind • A moving car • A running athelete • A rolling stone

10.JPG

• Flying aircraft

Formula for Kinetic Energy

If an object of mass ‘m’ moving with uniform velocity ‘u’, it is displaced through a distance ‘s’. Constant force ‘f’ acts on it in the direction of displacement. Its velocity changes from ‘u’ to ‘v’. Then acceleration is ‘a’.

Work done, W= f x s  &  f=ma

according to IIIrd kinematical equation, relationship between u,v,s and a is as follows:

V 2 -u 2 =2as

s = (v 2 -u 2 )/2a

Now putting the value of f and s , we have

W = (ma) x (v 2 -u 2 )/2a = m (v 2 -u 2 )/2

If u = 0 (when body starts moving from rest)

or, Kinetic Energy = ½mv 2

Example. An object of mass 15 kg is moving with uniform velocity of 4 m/sec. What is the kinetic energy possessed by it ?

Solution : Mass of the object, m = 15 kg Velocity of the object, v = 4 m/s K.E. = ½mv 2 = ½ × 15 kg × 4 m/s × 4 m/s = 120 J The kinetic energy of the object is 120 J.

Potential Energy

The energy of a body due to its position or change in shape is known as potential energy.

(i) Water kept in dam : It can rotate turbine to generate electricity due to its position above the ground.

(ii) Wound up spring of a toy car : It possess potential energy which is released during unwinding of spring. So toy car moves.

(iii) Bent string of bow : Potential energy due to change of its shape (deformation) released in the form of kinetic energy while shooting an arrow.

11.JPG

Factors affecting Potential Energy

(i) Mass : P. E. ∝ m More the mass of body, greater is the potential energy and vice-versa.

(ii) Height above the ground : P. E. ∝ h (Not depend on the path it follows) Greater the height above the ground, greater is the P.E. and vice-versa.

(iii) Change in shape : Greater the stretching, twisting or bending, more is the potential energy.

Potential Energy of an Object on a Height

If a body of mass ‘m’ is raised to a height ‘h’ above the surface of the earth, the gravitational pull of the earth (m × g) acts in downward direction. To lift the body, we have to do work against the force of gravity.

Thus, Work done, W = Force × Displacement Or W = m × g × h = mgh This work is stored in the body as potential energy (gravitational potential energy).

Thus, Potential energy, P.E. = m × g × h (where, g = acceleration due to gravity.)

12.JPG

Example. If a body of mass 10 kg is raised to a height of 6 m above the earth, calculate its potential energy.

Potential energy of the body = mgh Mass of body = 10 kg Height above the earth = 6 m Acceleration due to gravity = 10 m/s^2 So, P.E. = 10 × 10 × 6  = 600 J Thus, potential energy of the body is 600 Joules.

Transformation of Energy

The change of one form of energy to another form of energy is known as transformation of energy.

(i) A stone on a certain height has entire potential energy. But when it starts moving downward, potential energy of stone goes on decreasing as height goes on decreasing but its kinetic energy goes on increasing as velocity of stone goes on increasing. At the time stone reaches the ground, potential energy becomes zero and kinetic energy is maximum. Thus, its entire potential energy is transformed into kinetic energy.

(ii) At hydroelectric power house, the potential energy of water is transformed into kinetic energy and then into electrical energy.

(iii) At thermal power house, chemical energy of coal is changed into heat energy, which is futher converted into kinetic energy and electrical energy.

(iv) Plants use solar energy to make chemical energy in food by the process of photosynthesis.

Law of Conservation of Energy

• Whenever energy changes from one form to another form, the total amount of energy remains constant.

• “Energy can neither be created nor be destroyed.”

• Although some energy may be wasted during conversion, but the total energy of the system remains the same.

Conservation of Energy during Free Fall of a Body

• A ball of mass ‘m’ at a height ‘h’ has potential energy = mgh.

• As ball falls downwards, height ‘h’ decreases, so the potential energy also decreases.

• Kinetic energy at ‘h’ is zero but it is increasing during falling of ball.

• The sum of potential energy & kinetic energy of the ball remains the same at every point during its fall. ½mv2 + mgh = Constant Kinetic energy + Potential energy = Constant

Rate of Doing Work – Power

“Power is defined as the rate of energy consumption.”

Power = work done / time

or, P= W/t where,  P = Power W = Work done t = Time taken

Unit of Power SI unit of Power is Watt (W) = 1 Joule/second. 1 Watt = 1 J / 1 s

Power is 1 Watt when 1 joule work is done in one second.

Average Power = Total work done or total energy used / Total time taken

Power of Electrical Gadget

The power of an electrical appliance tells us the rate at which electrical energy is consumed by it.

Bigger unit of Power : Bigger unit of power is called Kilowatt or KW. 1 Kilowatt (KW) = 1000 Watt = 1000 W or 1000 J/s

Example. A body does 20 Joules of work in 5 seconds. What is its power ?

Power = Work done / Time taken Work done = 20 Joules Time taken = 5 sec. So, Power = 20 J / 5 s = 4 J/s = 4 W Thus, power of the body is 4 Watts.

Commercial Unit of Energy : Joule is very small unit of energy and it is inconvenient to use it where a large quantity of energy is involved.

For commercial purpose, bigger unit of energy is Kilotwatt hour (KWh).

1 KWh : 1 KWh is the amount of energy consumed when an electric appliance having a power rating of 1 Kilowatt is used for 1 hour.

Relation between Kilowatt hour and Joule

1 Kilowatt hour is the amount of energy consumed at the rate of 1 Kilowatt for 1 hour.

1 Kilowatt hour = 1 Kilowatt for 1 hour = 1000 Watt for 1 hour = 1000 Watt × 3600 seconds (60 × 60 seconds = 1 hour) = 36,00,000 Joules

So, 1 KWh = 3.6 × 106 J = 1 unit

Example. A bulb of 60 Watt is used for 6 hrs. daily. How many units (KWh) of electrical energy are consumed ?

Solution : Power of bulb = 60 W = 60 KW /1000 = 0.06 KW

t = 6 hours Energy = Power × Time taken = 0.06 × 6 h = 0.36 KWh = 0.36 units

Download CBSE Books

Exam Special Series:

  • Sample Question Paper for CBSE Class 10 Science (for 2024)
  • Sample Question Paper for CBSE Class 10 Maths (for 2024)
  • CBSE Most Repeated Questions for Class 10 Science Board Exams
  • CBSE Important Diagram Based Questions Class 10 Physics Board Exams
  • CBSE Important Numericals Class 10 Physics Board Exams
  • CBSE Practical Based Questions for Class 10 Science Board Exams
  • CBSE Important “Differentiate Between” Based Questions Class 10 Social Science
  • Sample Question Papers for CBSE Class 12 Physics (for 2024)
  • Sample Question Papers for CBSE Class 12 Chemistry (for 2024)
  • Sample Question Papers for CBSE Class 12 Maths (for 2024)
  • Sample Question Papers for CBSE Class 12 Biology (for 2024)
  • CBSE Important Diagrams & Graphs Asked in Board Exams Class 12 Physics
  • Master Organic Conversions CBSE Class 12 Chemistry Board Exams
  • CBSE Important Numericals Class 12 Physics Board Exams
  • CBSE Important Definitions Class 12 Physics Board Exams
  • CBSE Important Laws & Principles Class 12 Physics Board Exams
  • 10 Years CBSE Class 12 Chemistry Previous Year-Wise Solved Papers (2023-2024)
  • 10 Years CBSE Class 12 Physics Previous Year-Wise Solved Papers (2023-2024)
  • 10 Years CBSE Class 12 Maths Previous Year-Wise Solved Papers (2023-2024)
  • 10 Years CBSE Class 12 Biology Previous Year-Wise Solved Papers (2023-2024)
  • ICSE Important Numericals Class 10 Physics BOARD Exams (215 Numericals)
  • ICSE Important Figure Based Questions Class 10 Physics BOARD Exams (230 Questions)
  • ICSE Mole Concept and Stoichiometry Numericals Class 10 Chemistry (65 Numericals)
  • ICSE Reasoning Based Questions Class 10 Chemistry BOARD Exams (150 Qs)
  • ICSE Important Functions and Locations Based Questions Class 10 Biology
  • ICSE Reasoning Based Questions Class 10 Biology BOARD Exams (100 Qs)

✨ Join our Online JEE Test Series for 499/- Only (Web + App) for 1 Year

✨ Join our Online NEET Test Series for 499/- Only for 1 Year

Leave a Reply Cancel reply

Join our Online Test Series for CBSE, ICSE, JEE, NEET and Other Exams

Join Telegram Channel

Editable Study Materials for Your Institute - CBSE, ICSE, State Boards (Maharashtra & Karnataka), JEE, NEET, FOUNDATION, OLYMPIADS, PPTs

Discover more from Gurukul of Excellence

Subscribe now to keep reading and get access to the full archive.

Type your email…

Continue reading

Study Rankers

Extra Questions for Class 9th: Ch 11 Work and Energy Science

Extra questions for class 9th: ch 11 work and energy (science) important questions answer included.

case study based questions class 9 physics work and energy

Contact Form

IMAGES

  1. WORK AND ENERGY

    case study based questions class 9 physics work and energy

  2. CBSE Class 9 Science (Physics) Chapter 11 |Work and Energy

    case study based questions class 9 physics work and energy

  3. NCERT Solutions for Class 9 Science Chapter 11 Work and Energy in PDF

    case study based questions class 9 physics work and energy

  4. case study questions class 9 physics motion

    case study based questions class 9 physics work and energy

  5. NSTSE 2009 Solved Question Paper for Class IX

    case study based questions class 9 physics work and energy

  6. 9_Physics_Work and Energy(Work done by a constant force)_27.10.2020

    case study based questions class 9 physics work and energy

VIDEO

  1. Motion Class 9: Top 5 Case Study Based Questions

  2. Work and Energy : 25

  3. case study based questions for class 9 maths chapter 7(triangles)

  4. case study questions class 9 physics cbse science

  5. Top 20 MCQs from Work, Energy & Power

  6. MOST IMPORTANT QUESTIONS OF WORK ENERGY

COMMENTS

  1. Case Study Questions Class 9 Science

    CBSE Case Study Questions Class 9 Science - Work and Energy. (1) Work done by force acting on an object is equal to the magnitude of the force multiplied by the distance moved in the direction of the force. Work has only magnitude and no direction. Work done is negative when the force acts opposite to the direction of displacement.

  2. Class 9 Science Case Study Questions Chapter 11 Work and Energy

    Work and Energy Case Study Questions With Answers. Here, we have provided case-based/passage-based questions for Class 9 Science Chapter 11 Work and Energy. Case Study/Passage-Based Questions. Case Study 1: The figure shows a watch glass embedded in clay. A tiny spherical ball is placed at edge B at a height h above the center A.

  3. Case Study and Passage Based Questions for Class 9 Science Chapter 11

    Case Study Questions for Class 9 Science Chapter 11 Work and Energy. In CBSE Class 9 Science Paper, Students will have to answer some questions based on Assertion and Reason. There will be a few questions based on case studies and passage based as well. In that, a paragraph will be given, and then questions based on it will be asked.

  4. Class 9th Science

    Answers. <. Work and Energy Case Study Questions With Answer Key Answer Keys. Case Study. (i) (a) 0.05 J. The work done in raising the bob through a height of 5 cm (against the gravitational attraction) gets stored in the bob in the form of its potential energy. PE = mgh. = 0.1 x 10 x 5 x 10-2. = 0.05 J.

  5. Category: Case Study Questions for Class 9 Science

    Case Study and Passage Based Questions for Class 9 Science Chapter 11 Work and Energy. February 4, 2022 Physics Gurukul Leave a Comment.

  6. Top 5 case based questions from work & Energy

    Top 5 case based questions from work & Energy | CBSE Class 9 Physics ️ ⏮️ Subscribe to Vedantu 9 Channel to get Daily LIVE Classes Notifications👉 https://w...

  7. CBSE Class 9 Science Chapter 11

    Ans: The kinetic energy of an object is a kind of mechanical energy that exists in the object due to its state of motion (movement). 7. Write down the kinetic energy expression of an object. Ans: If m is the mass of an moving object and v is its velocity, then the expression of its kinetic energy (KE) will be.

  8. Class 9th Science

    Study Materials. Sep-09 , 2022. QB365 provides a detailed and simple solution for every Possible Case Study Questions in Class 9th Science Subject - Work and Energy, CBSE. It will help Students to get more practice questions, Students can Practice these question papers in addition to score best marks. E-mail.

  9. NCERT Solutions for Class 9 Science Chapter 11: Work and Energy

    NCERT Solutions Class 9 Science Chapter 11 - CBSE Free PDF Download *According to the CBSE Syllabus 2023-24, this chapter has been renumbered as Chapter 10. NCERT Solutions for Class 9 Science Chapter 11 Work and Energy help you lay a good foundation for your CBSE exam preparation. Students who refer to NCERT Solutions regularly are benefited from the comprehensive methodology and the ...

  10. NCERT Solutions for Class 9 Science Chapter 11 Work and Energy

    Answer: Battery converts chemical energy into electrical energy. This electrical energy is further converted into light and heat energy. Question 4: Certain force acting on a 20 kg mass changes its velocity from 5 m s -1 to 2 m s -1. Calculate the work done by the force. Answer: Mass of the body = 20 kg.

  11. CBSE Class 9 Physics Work and Energy Competency Based Questions

    Work and Energy - Competency Based Questions. Select the number of questions for the test: 5. 10. Strengthen your understanding of Work and Energy in CBSE Class 9 Physics through competency based questions. Acquire in-depth knowledge and improve problem-solving abilities with comprehensive solutions.

  12. Class 9 Science Case Study Questions

    If you are wondering how to solve class 9 science case study questions, ... As per the recent pattern of the Class 9 Science examination, a few questions based on case studies/passages will be included in the CBSE Class 9 Science Paper. There will be a paragraph presented, followed by questions based on it. ... Work, energy and power: Work done ...

  13. Work & Energy

    Learn. Commercial unit of electrical energy. Solved example - Cost of operation of electrical device. In this chapter, we will define the terms 'work' and 'energy'. We will learn how to calculate them, and use them to look at our world in a very different way.

  14. Case Study Questions of Chapter 11 Work and Energy PDF Download

    Here, we have provided case-based/passage-based questions for Class 9 Science Chapter 11 Work and Energy. Case Study/Passage Based Questions. Figure shows a watch glass embedded in clay. A tiny spherical ball is placed at the edge B at a height h above the centre A. The kinetic energy of the ball, when it reaches at point A is.

  15. Extra Questions for Class 9 Science Chapter 11 Work and Energy

    Calculate the work done by him on the luggage. Answer: (a) (i) Force should be applied. (ii) Body should move in the line of action of force. (iii) Angle between force and displacement should not be 90°. (b) Mass of luggage, m = 15 kg and displacement, s = 1.5 m. Work done, W = F×s = mg × s = 15 × 10 × 1.5 = 225 J.

  16. Class 9 Science Work & Energy (Questions from board papers)

    Question 1: Identify and state the type of transformation of energy in the following cases: (a) When coal is burnt. (b) In a thermal power plant [CBSE 2014] Answer 1: (a) Chemical energy stored in coal is converted to heat energy. (b) Chemical energy of fuel is first converted into heat energy than to kinetic energy and finally to electrical ...

  17. NCERT Solutions for Class 9 Science Chapter 11 Work and Energy

    Write an expression for the kinetic energy of an object. Answer. If a body of mass mis moving with a velocity v, then its kinetic energy E k is given by the expression, E k = 1/2 mv2. Its SI unit is Joule (J). 3. The kinetic energy of an object of mass, m moving with a velocity of 5 m s−1 is 25 J.

  18. Important Questions for CBSE Class 9 Science Work, Power and Energy

    Answer. The expression is KE = 1/2 mv 2, where 'm' is the mass and V is the velocity of the body. Question.25 Define 1 watt of power. Answer. When a work of 1 joule is done in 1s, the power is said to be one watt. Question.26 A lamp consumes 1000 J of electrical energy in 10 s.

  19. Work and Energy class 9 notes

    Here is Work and energy Class 9 Notes Summary. Work done is defined as the magnitude of the force multiplied by the distance moved by the object in the direction of the applied force. The unit of work is joule: 1 joule = 1 newton x 1 metre. An object having capability to do work is said to possess energy.

  20. NCERT solutions for Science Class 9 chapter 11

    The detailed, step-by-step solutions will help you understand the concepts better and clarify any confusion. NCERT solutions for Mathematics Science Class 9 CBSE 11 (Work and Energy) include all questions with answers and detailed explanations. This will clear students' doubts about questions and improve their application skills while preparing ...

  21. Important Questions for Class 9 Science Chapter 11 Work and Energy

    Important Questions for Class 9 Science Chapter 11 Work and Energy. Get important questions for Class 9 Science Chapter 11 Work and Energy with PDF. Our subject expert prepared these important questions and answers as per the latest NCERT textbook. These important questions will be helpful to revise the important topics and concepts. You can ...

  22. Work & Energy

    Potential Energy. The energy of a body due to its position or change in shape is known as potential energy. Examples : (i) Water kept in dam : It can rotate turbine to generate electricity due to its position above the ground. (ii) Wound up spring of a toy car : It possess potential energy which is released during unwinding of spring. So toy car moves.

  23. Extra Questions for Class 9th: Ch 11 Work and Energy Science

    Calculate the work done by him on the luggage. Answer. (a) (i) Force should be applied. (ii) Body should move in the line of action of force. (iii)Angle between force and displacement should not be 90°. (b) Mass of luggage, m = 15 kg and displacement, s = 1.5 m. Work done, W = F×s = mg × s = 15 × 10 × 1.5 = 225 J. Q4.